Download as pdf or txt
Download as pdf or txt
You are on page 1of 40

ABOUT DISHA PUBLICATION

One of the leading publishers in India, Disha Publication provides books and study
materials for schools and various competitive exams being continuously held across the
country. Disha’s sole purpose is to encourage a student to get the best out of preparation.
Disha Publication offers an online bookstore to help students buy exam books online with
ease. We, at Disha provide a wide array of Bank / Engg./ Medical & Other Competitive
Exam books to help all those aspirants who wish to crack their respective different levels
of Bank / Engg./ Medical & Other Competitive exams. At Disha Publication, we strive
to bring out the best guidebooks that students would find to be the most useful for all
kind of competitive exam.

DATA SUFFICIENCY
Topics No. of Questions Level
Concept Applicator 11 SNAP
Concept Builder 8 XAT
Concept Cracker 51 CAT
Concept Deviator 129 CAT, XAT advance level
Total 199

INTRODUCTION Tip 2 : Identify the parent question, means the main


question that is asked.
Data sufficiency is one of the most important as well
Tip 3 : Look at the statement (i) and information given
as most difficult type of question. Questions based
in that, see whether it is sufficient enough to
on data sufficiency requires deeper knowledge in the
answer the parent question.
subject area. Data sufficiency questions can be from
any topic viz Numbers, Arithmetic, Algebra, Geometry, Tip 4 : Look at the statement (ii) and information
and puzzles. These questions are not only difficult but given in that, most important is that when
also confusing. Students generally get confuse with the you see statement (ii) then just forget the
given information. Some general tips to solve the data information given in statement (i) , then see
sufficiency questions are discussed below. whether it is sufficient enough to answer the
parent question.
Tip 1 : 1st and foremost is understanding of options. In
Tip 5 : When the given statements are not sufficient
GMAT options are fixed but in other aptitude
then use information given in both the
test exams these options vary.
statements.
2

Directions (Qs. No. 1-7): Each of the following (I) P secured one-third marks of the total of Q, R
questions is followed by two statements. Mark and S.
[SNAP 2010] (II) Average marks obtained by Q and R are
(a) if statement I alone is sufficient to answer the 20 more than that secured by S.
question. 6. How many ice cubes can be accommodated in a
(b) if statement II alone is sufficient to answer the container?
question. (I) The length and breadth of the container is 20
(c) if both statement I and II together are necessary to cm and 15 cm respectively.
answer the question. (II) The edge of the ice cube is 2 cm.
(d) if both statements I and II together are not sufficient 7. Ram got ` 1500 as dividend from a company. What
to answer the question. is the rate of interest given by the company?
(I) The dividend paid last year was 10%.
1. Is ‘b’ positive?
(II) Ram has 350 shares of Rs 10 denomination.
(I) a + b is positive.
Directions (Qs. No. 8-11): Each of questions consist
(II) a – b is positive.
of question followed by two statements numbered
2. In a general body election, 3 candidates, p, q and r I and II [SNAP 2010]
were contesting for a membership of the board.
(a) if data in Statement I alone is sufficient to answer
How many votes did each receive?
the question but the data in Statement II alone is
(I) p received 17 votes more than q and 103 not sufficient to answer the question.
votes more than r.
(b) if data in Statement II alone is sufficient to answer
(II) Total votes cast were 1703. the question but the data in Statement I alone is not
3. If C1 and C2 are the circumferences of the outer sufficient to answer the question.
and inner circles respectively. What is C1 : C2? (c) if data in Statement I and II together are necessary
(I) The two circles are concentric. to answer the question.
(II) The area of the ring is 2/3 the area of greater (d) if data in Statement I and II together are not
circle. sufficient to answer the question.
4. What is the middle number of 7 consecutive 8. ∆ABC and ∆PQR are congruent
whole numbers? (I) Area of ∆ABC and ∆PQR are same
(I) Product of number is 702800. (II) ∆ABC and ∆PQR are right angle Triangles
(II) Sum of the number is 105. 9. Salary of A and B is in ratio 3:4 and expenditure is
5. Total marks obtained by P, Q, R and S in in ratio 4:5. What is the ratio of their saving?
Mathematics is 360. How many marks did P (I) B’s saving is 25% of his salary.
secure in Mathematics? (II) B’s salary is ` 2500.

Buy books : http://www.dishapublication.com/entrance-exams-books/mba-exams.html


3
10. What is the average height of the class? 11. Ram is taller than Shyam and Jay is shorter than
(I) Average height of the class decreases by 1 Vikram. Who is the shortest among them?
cm if we exclude the tallest person of the (I) Ram is the tallest.
class whose height is 56 cm.
(II) Shyam is taller than Vikram.
(II) Average height of the class increases by 1
cm if we exclude the shortest person of the
class whose height is 42 cm.

Directions (Qs. No. 1-2): are followed by two statements Directions (Qs. No. 3-4): are followed by two
labelled as I and II. Decide if these statements are statements labeled as I and II. You have to decide if
sufficient to conclusively answer the question. Choose these statements are sufficient to conclusively answer
the appropriate answer from the options given below: the question. Choose the appropriate answer from
[XAT 2011] options given below: [XAT 2008]
(a) Statement I alone is sufficient to answer the (a) If statement I alone is sufficient to answer the
question. question.
(b) Statement II alone is sufficient to answer the
(b) If statement II alone is sufficient to answer the
question.
question.
(c) Statement I and Statement II together are sufficient,
but neither of the two alone is sufficient to answer (c) If statement I and statement II together are
the question. sufficient but neither of the two alone is sufficient
(d) Either Statement I or Statement I alone is sufficient to answer the question.
to answer the question. (d) If either statement I or statement II alone is
(e) Neither Statement I nor Statement 11 is necessary sufficient to answer the question.
to answer the question.  (e) Both statement I and statement II are insufficient to
1. Let PQRS be a quadrilateral. Two circles 01 and answer the question.
02 are inscribed in triangles PQR and PSR 3. The base of a triangle is 60 cms, and one of the
respectively. Circle 01 touches PR at M and circle base angles is 60°. What is length of the shortest
02 touches PR at N. Find the length of MN. sideof the triangle?
I. A circle is inscribed in the quadrilateral (I) The sum of lengths of other two sides is 80
PQRS. cms.
II. The radii of the circles 01 and 02 are 5and 6 (II) The other base angle is 45°.
units respectively. [5]
4. A, B, C, D, E and F are six integers such that E <
2. Given below is an equation where the letters
F, B > A, A < D < B. C is the greatest integer. Is A
represent digits. (PQ). (RQ) = XXX. Determine
the smallest integer?
the sum of P + Q + R + X.
I. X = 9. (I) E+B<A+D
II. The digits are unique. [3] (II) D<F
Buy books : http://www.dishapublication.com/entrance-exams-books/mba-exams.html
4
Directions (Qs. No. 5-8): are followed by two 6. ABC is a triangle with ∠B = 90°. What is the
statements labelled as (1) and (2). You have to decide length of the side AC?
if thesestatements are sufficient to conclusively answer 1. D is the midpoint of BC and E is the midpoint
the question. Choose [XAT 2007] of AB.
(a) If statement (1) alone is sufficient to answer the 2. AD = 7 and CE = 5
question. 7. Five integers A, B, C, D and E are arranged in such
(b) If statement (2) alone is sufficient to answer the a way that there are two integers between B and C
question. and B is not the greatest. There exists one integer
(c) If statement (1) and Statement (2) together are between D and E and D is smaller than E. A is not
sufficient but neither of the two alone is sufficient the smallest integer. Which one is the smallest?
to answer the question. 1. E is the greatest
(d) If either statement (1) or Statement (2) alone is 2. There exists no integer between B and E.
sufficient to answer the question. 8. Le f : N → N (N is the set of all natural numbers).
(e) Both statement (1) and statement (2) are insufficient How many solutions are there to the equation f (x)
to answer the question. = 1485 ?
5. What is he maximum value of a/b? 1. For a, b ∈N f(10a + b) = f(a) + 12b
1. a, a + b and a + 2b are three sides of a triangle. 2. The maximum value of b is 9
2. a and b both are positive.

Directions (Qs. No. 1-10): Each question is followed B.


Exclusion of the heaviest and the lightest
by two statements, A and B. Answer each question members from the class does not change the
using the following instructions: [CAT 1999] average weight of the students. 
2. A small storage tank is spherical tank in shape.
(a) if the question can be answered by using one of the
What is the storage volume of the tank?
statements alone, but cannot be answered using
 A. The wall thickness of the tank is 1cm.
the other statement alone.  B. When the empty spherical tank is immersed
(b) if the question can be answered by using either in a large tank filled with   water, 20litres of
statement alone. water overflow from the large tank.
(c) if the question can be answered by using both 3. Mr. X starts walking northwards along the
statements together, but cannot be answered using boundary of a field, from point A on the boundary,
either statement alone. and after walking for 150 metres reaches B, and
(d) if the question cannot be answered even by using then walks westwards, again along the boundary,
both statements together. for another 100 metres when he reaches C. What is
the maximum distance between any pair of points
1. The average weight of students in class is 50 kg..
on the boundary of the field?
What is the number of students in the class? A.   The field is rectangular in shape.  
A. The heaviest and the highest members of the B.    The field is a polygon, with C as one of its
class weigh 60 kg and 40 kg respectively. vertices and A the mid point of a side.
Buy books : http://www.dishapublication.com/entrance-exams-books/mba-exams.html
5
4. A line graph on a graph sheet shows the revenue 9. What is the distance x between two cities A and B
for each year from 1990 through 1998 by points in integral number of Kms?
and joins the successive points by straight line A. x satisfies the equation log2x =  x    
segments. The point for revenue of 1990 is labelled
B. x < 10 Kms
A, that for 1991 as B, and that for 1992 as C. What
is the ratio of growth in revenue between 91-92 10 Mr. Mendel grew one hundred flowering plants
and 90-91? from black seeds and white seeds, each seed giving
rise to one plant. A plant gives flowers of only one
A. The angle between AB and X-axis when
colour. From a black seed comes a plant giving red
measured with a protractor is 40 degrees,
or blue flowers. From a white seed comes a plant
and the angle between CB and X-axis is 80
giving red or white flowers. How many black seeds
degrees.
were used by Mr. Mendel?  
B. The scale of Y-axis is 1cm = 1000 Rs.
A. The number of plants with white flowers was
5. There is a circle with centre C at the origin and
10.
radius r cm. Two tangents are drawn from an
external point D at a distance d cm from the centre. B. The number of plants with red flowers was
What are the angles between each tangent and the 70.
X-axis? Directions (Qs. No. 11-20): Each question is followed
A.   The coordinates of D are given by two statements, A and B. Answer each question
B.   The X-axis bisects one of the tangents. using the following instructions: [CAT 2000]
6. Find a pair of real numbers x and y that satisfy (a) if the question can be answered by using one of the
the following two equations simultaneously. It is statements alone, but cannot be answered using the
known that the values of a, b, c, d, e and f are non- other statement alone.
zero. (b) if the question can be answered by using either
ax + by = c statement alone.
dx + ey = f (c) if the question can be answered by using both
A. a = kd and b = ke, c = kf, k ≠ 0 statements together, but cannot be answered using
B. a = b = 1, d = e = 2, f ≠ 2c either statement alone.
7. Three professors A, B and C are separately (d) if the question cannot be answered even by using
given three sets of numbers to add. They were both statements together.
expected to find the answers to 1+1, 1+1+2, and 11. Consider three real numbers, X, Y and Z. Is Z the
1+1 respectively. Their respective answers were smallest of these numbers?
3, 3, and (2) How many of the professors are
A. X is greater than at least one of Y and Z.
mathematicians?
B. Y is greater than at least one of X and Z.
A. A mathematician can never add  two numbers
correctly, but can always add three numbers 12. Let X be  a real number. Is the modulus of X
correctly. necessarily less than 3?
B. When a mathematician makes a mistake in a A. X (X + 3) < 0
sum, the error is +1 or – (1) B. X (X – 3) > 0
8. How many among the four students A, B, C and D 13. How many people are watching TV programme P?
have passed the exam? A. Number of people watching TV programme
 A. The  following is a true statement : A and B Q is 1000 and number of people watching
passed the exam. both the programmes, P and Q, is 100.
B The following is a false statement. At least B. Number of people watching either P or Q or
one among C and D has passed the exam.   both is 1500.
Buy books : http://www.dishapublication.com/entrance-exams-books/mba-exams.html
6
14. Triangle PQR has angle PRQ equal to 90°, what is A. The average speed of the plane is 700
the value of PR + RQ? kilometres per hour.
A. Diameter of the inscribed circle of the triangle B. The flight distance is 10,500 kilometres.
PQR is equal to 10 cm. 20.   What are the ages of two individuals, X and Y?
B. Diameter of the circumscribed circle of the A.      The age difference between them is 6 years.
triangle PQR is equal to 18 cm. B.      The product of their ages is divisible by 6.
15. Harshad bought shares of a company on a certain Directions (Qs. No. 21-27): Each item is followed by
day, and sold them the next day. While buying and two statements, A and B. Answer each question using
selling he had to pay to the broker one percent of the following instructions: [CAT 2001]
the transaction value of the shares as brokerage.
What was the profit earned by him per rupee spent (a) if the question can be answered by one of the
on buying the shares? statements alone and not by the other.
A. The sales price per share was 1.05 times that (b) Choose b   if the question can be answered by using
of its purchase price. either statement alone.
(c) Choose c if the question can be answered by
B. The number of shares purchased was 100.
using both the statements together, but cannot be
16. For any two real numbers:
answered by using either statement alone.
a + b = 1 if both a and b are positive or both a and (d) Choose d if the question cannot be answered even
b are negative. by using both the statements together.
=  –1 if one of the two numbers  a and  b is 21. What are the value of m and n?
positive and the other negative. A. n is an even integer, m is an odd and m is
What is (2 + 0) Å (–5 + –6)? greater than n.
A. a + b is zero if a is zero B. Product of m and n is 30.
B. a + b = b + a 22. Is Country X’s GDP higher than country Y’s GDP?
17. There are two straight lines in the x-y plane with A. GDP’s of the countries X and Y have grown
equations: over the past 5 years at compounded annual
ax + by = c rate of 5% and 6% respectively.
dx + ey = f B. 5 years ago, GDP of country X was higher
Do the two straight lines intersect? than that of country Y.
A. a, b, c, d, e and f are distinct real numbers. 23. What is the value of X?
B. c and f are non-zero  A. X and Y are unequal even integers less than
18. O is the centre of 2 concentric circle. ae  is a chord 10 and X/Y is an odd integer.
of the outer circle and it intersects the inner circle  B. X and Y are even integers each less than 10
at points b and d. C is a point on the chord in and product of X and Y is 12.
between b and d. What is the value of ac/ce? 24. On a given day a boat ferried 1500 passengers
A. bc/cd = 1 across the river in 12 hrs. How many round trips
did it make?
B. A third circle intersects the inner circle at b
and d and the point c is on the line joining the A. The boat can carry 200 passengers at any
centres of the third circle and the inner circle. time.
B. It takes 40 minutes each way and 20 minutes
19. Ghosh Babu has decided to take a non-stop flight
for waiting time at each terminal.
from Mumbai to No-man’s-land in South America.
He is scheduled to leave Mumbai at 5 am, Indian 25. What will be the time for downloading software?
Standard Time on December 10, 2000. What is the A. Transfer rate is 6 kilobytes per second.
local time at No-man’s-land when he reaches there? B. The size of the software is 4.5 megabytes.
Buy books : http://www.dishapublication.com/entrance-exams-books/mba-exams.html
7
26. A square is inscribed in a circle. What is the A. After reducing the listed price by 10 percent,
difference between the area of the circle and that the dress sold for a net profit of 10 dollars.
of the square? B. The dress sold for 50 dollars.
A. The diameter of the circle is 25 2 cm. 32. Is 500 the average(arithmetic mean) score on the
B. The side of the square is 25 cm. GMAT?
27. Two friends, Ram and Gopal, bought apples from a A. Half of the people who take the GMAT score
wholesale dealer. How many apples did they buy? above 500 and half of the people score below
A. Ram bought one-half the number of apples 500.
that Gopal bought. B. The highest GMAT score is 800 and the
B. The wholesale dealer had a stock of 500 lowest score is 200.
apples. 33. Is |x – 2| < 1?
A. |x| > 1
Directions (Qs. No. 28-35): Each item is followed by B. |x – 1| < 2
two statements, A and B. Answer each questions using 34. People in a club either speak French or Russian
the following instructions. [CAT 2002] or both. Find the number of people in a club who
(a) if the question can be answered by one of the speak only French.
statements alone but not bythe other. A. There are 300 people in the club and the
(b) if the question can be answered by using either number of people who speak both French and
statement alone. Russian is 196.
(c) if the question can be answered by using both B. The number of people who speak only
thestatements together, but cannot be answered by Russian is 58.
us either statement alone. 35. A sum of ` 38,500 was divided among Jagdish,
(d) if the question cannot be answered by either of the Punit and Girish. Who received the minimum
statements. amount?
28. In a hockey match, the Indian team was behind by A. Jagdish received 2/9 of what Punit and Girish
2 goals with 5 minutes remaining. Did they win the together received.
match? B. Punit received 3/11 of what Jagdish and
A. Deepak Thakur, the Indian striker, scored 3 Girish together received.
goals in the last five minutes of the match.
Directions (Qs. No. 36-39): In each question there are
B. Korea scored a total of 3 goals in the match.
29. Four students were added to a dance class. Wouldtwo statements A and B
(a) if the question can be answered by one of the
the teacher be able to divide her students evenly
into a dance team (or teams) of 8? statements alone but not by the other.
A. If 12 students were added, the teacher could(b) if the question can be answered by using either
put everyone in teams of 8 without any statement alone.
leftovers. (c) if the question can be answered by using both the
B. The number of students in the class is statements together but cannot be answered using
currently not divisible by 8. either statement alone.
30. Is x = y ? (d) if the question cannot be answered even by using
A. (x + y)( 1 / x + 1 / y) = 4 both the statements A and B. [CAT 2003]
B. (x – 50) 2 = ( y – 50) 2 36. F and M are father and mother of S, respectively. S
31. A dress was initially listed at a price that would has four uncles and three aunts. F has two siblings.
have given the store a profit of 20 percent of the The siblings of F and M are unmarried. How many
wholesale cost. What was the wholesale cost of the brothers does M have?
dress? A. F has two brothers.
B. M has five siblings.
Buy books : http://www.dishapublication.com/entrance-exams-books/mba-exams.html
8
37. A game consists of tossing a coin successively. There
40. Four candidates for an award obtain distinct scores
is an entry fee of `10 and an additional fee of ` 1 in a test. Each of the four casts a vote to choose the
for each toss of the coin. The game is considered winner of the award. The candidate who gets the
to have ended normally when the coin turns heads largest number of votes wins the award. In case
on two consecutive throws. In this case the player of a tie in the voting process, the candidate with
is paid `100. Alternatively, the player can choose the highest score wins the award. Who wins the
to terminate the game prematurely after any of the award?
tosses. Ram has incurred a loss of ` 50 by playing A: The candidates with top three scores each vote
this game. How many times did he toss the coin? for the top scorer amongst the other three.
A. The game ended normally. B: The candidate with the lowest score votes for
B. The total number of tails obtained in the the player with the second highest score.
game was 138. 41. Zakib spends 30% of his income on his children’s
38. Each packet of SOAP costs ` 10. Inside each packet education, 20% on recreation and 10% on
is a gift coupon labelled with one of the letters S, O, healthcare. The corresponding percentages for
A and P. If a customer  submits four such coupons Supriyo are 40%, 25%, and 13%. Who spends
that make up the word SOAP, the customer gets more on children’s education?
a free SOAP packet. Ms. X kept buying packet A: Zakib spends more on recreation than
after packet of SOAP till she could get one set of Supriyo.
coupons that formed the word SOAP. How many B: Supriyo spends more on healthcare than Zakib.
coupons with label P did she get in the above 42. Tarak is standing 2 steps to the left of a red mark
process? and 3 steps to the right of a blue mark. He tosses
A. The last label obtained by her was S and the a coin. If it comes up heads, he moves one step to
total amount spent was ` 210 the right; otherwise he moves one step to the left.
B. The total number of vowels obtained was 18. He keeps doing this until he reaches one of the two
39. If A and B run a race, then A wins by 60 seconds. marks, and then he stops. At which mark does he
If B and C run the same race, then B wins by 30 stop?
seconds. Assuming that C maintains a uniform A: He stops after 21 coins tosses.
speed what is the time taken by C to finish the B: He obtains three more tails than heads.
race? 43. In a class of 30 students, Rashmi secured the third
A. A and C run the same race and A wins by 375 rank among the girls, while her brother Kumar
metres studying in the same class secured the sixth rank in
B. The length of the race is 1 km. the whole class. Between the two, who had a better
overall rank?
Directions (Qs. No. 40-45): Each question is followed
A: Kumar was among the top 25% of the boys
by two statements, A and B. Answer each question
merit list in the class in which 60% were boys.
using the following instructions: [CAT 2004]
B: There were three boys among the top five
(1) if the question can be answered by using one of rank holders, and three girls among the top
the statements alone but not by using the other ten rank holders.
statement alone. 44. Nandini paid for an article using currency notes of
(2) if the question can be answered by using either of denominations ` 1, ` 2, ` 5 and ` 10 using at least
the statements alone. one note of each denomination. The total number
(3) if the question can be answered by using both of five and ten rupee notes used was one more than
statements together but not by either statement the total number of one and two rupee notes used.
alone. What was the price of the article?
(4) if the question cannot be answered on the basis of A: Nandini used a total of 13 currency notes.
the two statements. B: The price of the article was a multiple of ` 10.
Buy books : http://www.dishapublication.com/entrance-exams-books/mba-exams.html
9
45. Ravi spent less than ` 75 to buy one kilogram each 48. In a football match, at the half-time, Mahindra and
of potato, onion, and gourd. Which one of the three Mahindra Club was trailing by three goals. Did it
vegetables bought was the costliest? win the match?
A: 2 kg potato and 1 kg gourd cost less than 1 kg A. In the second-half Mahindra and Mahindra
potato and 2 kg gourd. Club scored four goals.
B: 1 kg potato and 2 kg onion together cost the B. The opponent scored four goals in the match.
same as 1 kg onion and 2 kg gourd. 49. In a particular school, sixty students were athletes.
Ten among them were also among the top academic
Directions (Qs. No. 46-49): Each question is followed performers. How many top academic performers
by two statements, A and B. Answer each question were in the school?
using the following instructions: [CAT 2007] A. Sixty per cent of the top academic performers
(a) If the question can be answered by using the were not athletes.
statement A alone but not by using the statement B B. All the top academic performers were not
alone. necessarily athletes.
(b) If the question can be answered by using the Directions (Qs. No. 50-51): [CAT 2008]
statement B alone but not by using the statement A (a) if Question can be answered from A alone but not
alone. from B alone.
(b) if Question can be answered from B alone but not
(c) If the question can be answered by using either of
from A alone.
the statements alone.
(c) if Question can be answered from A alone as well
(d) If the question can be answered by using both
as from B alone.
the statements together but not by either of the
(d) if Question can be answered from A and B together
statements alone. but not from any of them alone.
(e) If the question cannot be answered on the basis of (e) if Question cannot be answered even from A and B
the two statements. together.
46. Five students Atul, Bala, Chetan, Dev and Ernesto In a single elimination tournament, any player is
were the only ones who participated in a quiz eliminated with a single loss. The tournament is played
contest. They were ranked based on their scores in in multiple rounds subject to the following rules:
the contest. Dev got a highet rank as compared to (i) If the number of players, say n, in any round is
Ernesto, while Bala got a higher rank as compared even, then the players are grouped in to n/2 pairs.
to Chetan. Chetan’s rank was lower than the The players in each pair play a match against each
other and the winner moves on to the next round.
median. Who among the five got the highest rank?
(ii) If the number of players, say n, in any round
A. Atul was the last rank holder.
is odd, then one of them is given a bye, that is,
B. Bala was not among the top two rank holders. he automatically moves on to the next round.
47. Thrity per cent of the employees of a call centre are The remaining (n − 1) players are grouped into
males. Ten per cent of the female employees have (n – 1)/2 pairs. The players in each pair play a
an engineering background. What is the percentage match against each other and the winner moves on
of male employees with engineering background? to the next round. No player gets more than one
A. Twenty five per cent of the employees have bye in the entire tournament.
engineering background. Thus, if n is even, then n/2 players move on to the next
B. Number of male employees having an round while if n is odd, then (n + 1)/2 players move on
engineering background is 20% more than to the next round. The process is continued till the final
the number of female employees having an round, which obviously is played between two players.
engineering background. The winner in the final round is the champion of the
tournament
Buy books : http://www.dishapublication.com/entrance-exams-books/mba-exams.html
10
49. What is the number of matches played by the 50. If the number of players, say n, in the first round
champion? was between 65 and 128, then what is the exact
A: The entry list for the tournament consists of value of n?
83 players. A: Exactly one player received a bye in the
B: The champion received one bye. entire tournament.
B: One player received a bye while moving on
to the fourth round from third round

Directions for Data Sufficiency 6. Is P + Q a rational number?


(a) If statement 1 alone is sufficient to answer the (1) P and Q both irrational
question, but statement 2 alone is not sufficient (2) PQ is rational
(b) If statement 2 alone is sufficient to answer the 7. When 0.abc is written in the form of P/Q then is
question, but statement 1 alone is not sufficient P + Q divisible by 9?
(c) If both statements together are needed to answer the (1) Three digit number “abc” is divisible by 9
question, but neither statement alone is sufficient
(2) Any two digit number formed by digits a, b
(d) If either statement by itself is sufficient to answer
or c is divisible by 9.
the question
8. Is (a^b + b^a) positive number?
(e) If not enough facts are given to answer the question
(1) a + b is positive.
1. Is K an integer?
(2) ab is positive
(1) (PK) is a positive integer where P is a prime
9. Is four digit number abcd divisible by 4?
number.
(1) abcd is divisible by 1st 4 prime numbers.
(2) (K/P) is a positive integer
(2) The four digit number “abcd” is divisible by
2. If A and B (B ≠ 0) are integers, is A/B an integer?
n^3-n^2 where n is odd and more than 77.
(1) A is an even integer and B is a prime number
10. Is a + b + c a two digit number.
BA
(2) is an integer. (1) all a, b and c is more than 3 but less than 30
AB (2) if 7 < a < b < c < 11
3. If N is a perfect square less than 1000 what is the
value of N? 11. Is three digit number N divisible by 9?
(1) N is odd. (1) All the digits in N is distinct and sum and
(2) N is a perfect cube. product of digits is 6
4. Is abc + def is an even number (2) N is divisible by only 1st 4 prime numbers.
(1) abc def + def abc is an even number 12. If a number N = a.bcd when written in the form
(2) a^b^c + d^e^f is an even number P/Q then is P-Q divisible by 9
5. Is a four digit number “abcd” divisible by 9 (1) Two digit number ‘dc’ is divisible by 9.
(1) (a^b)(b^c) divisible by 9 (2) When two digit number “dc” divided by 6
resultant is not divisible by 3.
(2) (a + b)(c + d) is divisible by 9
Buy books : http://www.dishapublication.com/entrance-exams-books/mba-exams.html
11
13. Is number P a prime number if its square root is K. 22. Is number N2 has more than 35 factors?
abcd…… (1) N has 12 factors
(1) P is not divisible by any prime number less (2) In 4 ways we can write N as product of two of
than K its co-prime factors.
(2) P is not divisible by any prime number more 23. Out of three non zero digits A, B and C which one
than K has highest number of factors?
14. Is A + B a rational number? (1) If A is prime then B and C is perfect square.
(1) A is sum of square roots of two prime (2) If B is odd then A and C are even.
numbers 24. How many numbers are less than N and co prime
(2) B is difference of square roots of two prime to N?
numbers (1) N is a two digit prime number with sum of
15. Is product of three numbers a, b and c a positive digits 16
integer? (2) N is a two digit prime number with product
(1) At least 2 of three is a positive integer. of digits 42
(2) At least 1 of the three are negative integer. 25. Is N/6 has odd number of factors?
16. Is product of three numbers a, b and c a positive (1) N/24 has odd number of factors
even number? (2) 24 N has odd number of factors
(1) At least 2 of the three is an even positive 26. If N has 81 factors then how many factors does N3
integer. has.
(2) At most 1 of the three is an even negative (1) In 4 ways we can write N14 as product of 2 of
integer. its co-prime factors.
17. What is minimum value of A/B + B/A? (2) N2 has maximum possible number of factors
(1) The product AB > 0 27. Is number of factors of N divisible by 2 more than
(2) The sum A +B > 0 that divisible by 3
18. Is ab + bc + ca positive? (1) N is not divisible by 24 but is divisible by 28.
(1) At least two of the three given numbers is (2) Nis not divisible by 54 but is divisible by 63
positive 28. If N has 81 factors then find the number of factors
(2) At least one of them is negative of 2N
19. Is product of 5 consecutive two digit natural (1) N is not divisible by 24 but is divisible by 28.
numbers divisible by 25? (2) N is not divisible by 54 but is divisible by 63.
(1) Sum of those 5 natural numbers is more than 29. If N has 16 factors then find the value of N
239 (1) N is divisible by 64.
(2) Sum of those 5 natural numbers is less than (2) N is divisible by 72.
260 30. What is the ratio of number of factors of N2 to that
20. A “centum set a” is define as set of numbers whose of N
th (1) N 3 has 112 factors
100 digit is a. Find the number of such sets.
(1) If all the members of the set is 3 digit numbers (2) Number of factors of N2 is more than 64 but
and any member of the set has only prime less than 81.
digits. 31. Find the number of factors of N
(2) If all the members of the set is a 5 digit (1) N/2 has 12 factors
number. (2) N is a perfect cube
21. Is number N has odd number of factors? 32. N has 18 factors is it divisible by 18?
(1) N is divisible by only one prime number. (1) N is divisible by 32 but not by 512
(2) 61 < N < 67 (2) N is divisible by 5
Buy books : http://www.dishapublication.com/entrance-exams-books/mba-exams.html
12
33. Find the number of factors of N 45. What is remainder when N is divided by 9?
(1) P = NQ (1) N give remainder 1 when it is divided by 3 or 4.
(2) Number of factors of P = 6Q2+5Q +1 (2) N when successively divided by 3, 4, 5 and 6
34. Find the number of odd factors of N it leads remainder 1 in every step.
(1) Number of even factors of N is 17 46. Is ratio of LCM to HCF of two numbers more than 8?
(2) 2N has 17 odd factors (1) One of the numbers is 5 and their LCM is 40.
35. How many factors of number N has unit digit 5 if (2) HCF of two numbers is 5 and one of the
N is divisible by 5. number is 15
(1) N has 21 odd factors 47. Is (ABC) and (DEF) co-prime to each other?
(1) HCF of (AB), (CD) and (EF) is 1.
(2) None of the factors divisible by 10.
(2) HCF of (A and D), (B and E) and (C and F) in
36. How many ways N can be written as product of 2
pairs is 1.
of its co-prime factors?
48. Find the value of f (20) if f (n+2) is LCM of f (n)
(1) Only single digit prime numbers divide N.
and f (n+1) here n is a natural number.
(2) N is divisible by 9 but not by 10. (1) f (7) = 36
37. Is number of even factors of N more than number (2) f (1) = 1
of odd factors? 49. Is HCF of a and b more than 1, here a and b are
(1) Given that number is divisible by 10 distinct positive integers?
(2) Number of factors of N is odd (1) a is divisible by 9 but not by 18
38. Is integer k > 5 ? (2) b is divisible by a2 but not by a2
(1) K is a factor of 30. 50. Is A & B co-prime to each other ?
(2) K is not a factor of 120 (1) LCM of A & B is 36 and one of the number is
39. How many factors of number N is a two digit prime.
number? (2) Number of factors of their HCF is 1.
(1) N has total number of factors as 16. 51. What is the number of students in class 8? If number
(2) N has 4 single digit prime factors. of students in a class cannot be more than 100.
40. Is number of factors of N more than 27? (1) When students are divided in a group of 4
(1) N is a perfect square with at least 3 prime only 2 students are left but when divided in a
factors. group of 11, only 9 students are left.
(2) If N is a perfect cube then it is a perfect (2) When students are divided in a group of 5
square also. only 2 students are left but when divided in a
41. Is A > B if their LCM is 30. group of 7, no student is left.
52. How many days in the month March both Maths
(1) HCF of A and B is 1 and when (a + b) divided
and English faculty will come together?
by 9 remainder is 4.
(1) Maths faculty comes at an interval of 3 days
(2) AB is odd.
and English faculty at an interval of 4 days.
42. What is HCF of (A + B) and (A – B) given that A is
Mathsfaculty came on 3rd, 6th, 9th January
not equal to B? and so on.
(1) HCF of A and B is 4 English faculty came on 4th, 8th, 12th, January
(2) LCM of A and B is 36 and so on.
43. Is HCF of A and B more than 4? (2) Maths faculty comes at an interval of 3 days
(1) LCM of A and B is 48. and English faculty at an interval of 7 days.
(2) The ratio of A:B is 1;2 Maths faculty came on 3rd, 6th, 9th January
44. Are A and B co-prime to each other? and so on.
(1) AB and B are co-prime to each other. English faculty came on 7th, 14th, 21st,
B
(2) HCF of A and B is 1A January and so on.
Buy books : http://www.dishapublication.com/entrance-exams-books/mba-exams.html
13
53. If H1 is the HCF of (11111….P times) and 61. Find the unit digit of xy
(333333…… Q times) and H2 is the HCF of (1) x and y are prime digits
(2222…. R times) and (5555…. S times) is H1 > H2? (2) unit digit of yx is 0
(1) HCF of P & Q is 5, P & R is 7 and P & S is 14. 62. What is the unit digit of 7abcd
(2) HCF of P & Q is 5 and R & S is 7. (1) Four digit number cbad is divisible by 4.
Consider Q > P then H1 = 11111….P times, but if
(2) When 4 digit number ‘abcd’ divided by 72 it
P > Q then H1 = can be 1111… Q times or
leaves remainder 1.
54. Is R > 12 x
(1) When 252, 612 and 1212 divided by N 63. What is the unit digit of 33 .
remainder is R in each case. (1) x is a prime number.
(2) When 255, 612 and 1215 divided by N, (2) x is a three digit prime number
remainder is R in each case.
x x −1
55. Is HCF of A, B, C and D more than 5. 64. What is the unit digit of 4
(1) HCF of A & B is 20 and HCF of C and D is 35 (1) (x – 1) has only 2 factors.
(2) HCF of A & B is 10, A & C is 20 and A & D (2) (x + 1) has only 2 factors.
is 30 d
bc
56. Is HCF of A, B, C and D more than 5 if they are 65. What is the unit digit of 2a
distinct non-prime number? (1) All the numbers a, b, c and d are even
(1) LCM of A & B is 50 and that of B & C is 25 numbers.
(2) LCM of A& C is 50 and that of B & D is 75. (2) All the numbers a, b, c and d are odd numbers.
57. What is remainder when (x + y + z) divided by 3 if
cd
x, y and z are distinct positive integers? ab
66. What is the unit digit of 3
(1) A number gives remainder x, y and z when
divided by 4, 8 and 16 respectively. (1) All the numbers a, b, c and d are prime
(2) HCF of xy, yz and zx is 3. numbers.
58. Is HCF of A. B and C one? (2) All the numbers a, b, c and d are three digit
(1) HCF of AB, BC, and CA (i.e product of two numbers.
numbers taken at a time) is 1 67. If unit digit of ab is 1 then what is the value of a?
(2) HCF of (AB)C, (BC)A, and (AC)B is 1. (1) b is even number
59. If F(3k) = Product of F(3k – 2) and F(3k – 1); (2) a is divisible by 9
F(3k + 1) = LCM of F(3k – 1) and F(3k); 68. Find the unit digit of a 11111
F(3k + 2) = HCF of F(3k) and F(3k + 1) here k is a (1) Unit digit of a66 is 1
natural number. (2) Unit digit of a999 is 9
b
Find the HCF of F(3n), F(3n + 3), F(3n + 6) …. 69. Find the unit digit of a
333
(1) Unit digit of a is 9
F(3n + 3p)
(1) F(3n+9) = 12 48 (2) Unit digit of a555 is 6
x y
(2) HCF of F(3n – 6) and F(3p – 6) is 1224 if p > n 70. Find the unit digit of 9 + 9 .
60. If A = a4444 – 1 and B = a4444 + 1 find unit digit of (1) x + y is odd
LCM of A and B. (2) (x.y) is odd
2
(1) When a divided by 8 remainder is 1. 71. What is the unit digit of (xy)xy ? Here “xy” and
(2) a can be expressed as difference between “yx” is a two-digit number.
squares of 2 consecutive natural numbers, (1) Two digit number xy is divisible by 4.
99
and a is not divisible by 625 (2) (xy)xy is not divisible by 100.
Buy books : http://www.dishapublication.com/entrance-exams-books/mba-exams.html
14
72. If unit digit of 3x + 3y is 0 then find the unit digit of  5x 
3xy 82. Find the remainder of  
9
(1) 7x has unit digit either 1 or 7 (1) x is a 2 digit prime no.
(2) 2y has unit digit either 4 or 8 (2) x when divided by 12 gives remainder 9.
73. Find the unit digit of 3x – 3y  a 7777 
(1) Both x is in the form of 4p +1 while y is in the 83. What is the remainder of  
 7 
form of4p + 3 here p & q are positive integers. (1) given that a is in the form of 5x + 1
(2) x when divided by 4 gives remainder 1 and (2) a is in the form of 4x + 1
when y divided by 4 gives remainder 3  ab 
74. Find the unit digit of (x7)(7x) 7
84. What is the remainder of   here a, b are
(1) Unit digit of x7 + 7x is 6 distinct integers.  9 
(2) Unit digit of x7 + 7x is 8 (1) a, b both are prime numbers.
75. Find the unit digit of x y + yx (2) a, b both are even integers.
(1) Unit digit of x y is 3  xy 
85. What is the remainder of  
(2) Unit digit of y x is 1.  z 
(1) |x – z| = 1
76. Find the Last two digit of 2x
 x3 
(1) x gives remainder 4 and 5 when divided by 5 (2) Remainder of   is 1
or 6.  z 
(2) x gives remainder 3 and 4 when divided by 4  x
86. Is remainder of   is 1
or 5. z
77. Find the last two digit of (8181)xyz here xyz is a  x ^ 2
(1) Remainder of  z  is 1
three digit number.  
(1) Three digit number ‘xyz’ is divisible by 5  x3 
(2) Remainder of  z  is 1
and 7  
(2) Three digit number ‘xyz’ is divisible by 5  3x 
and 23 87. If R1 is remainder of   and R2 is the remainder
7
78. Find the last two digits of a^b^c^d^e.  3y 
of   is R1>R2 here x and y both are positive
(1) Last two digit of a^b is 76 7
(2) Last two digits of a ^b is 12 integers.
79. What is the last two digit of (ab)ba here ‘ab’ and (1) x = 6y
‘ba’ are two digit numbers and difference between (2) x = 4y
 2 x3 y 
ab and ba is not divisible by 27. 88. What is the remainder of  
 7 
(1) ab has unit digit 1
(1) x and y both divisible by 3.
(2) ba has unit digit 4
(2) both x and y are odd.
80. Find the last two digits of (aa)bb  yz  
(1) Last two digits of ab is 29   x  
89. What is the remainder of    ?
(2) Last two digits of ab is 56
 xy 
 3x  x
81. What is the remainder of  (1) Remainder of   is 1.
  y
7
(1) x is in the form of( 4k + 2)  y
(2) Remainder of   is 5
(2) x is in the form of (3k + 1) x
Buy books : http://www.dishapublication.com/entrance-exams-books/mba-exams.html
15
 bc 
90. What is the remainder of   here ‘xyz’ is a
xyz
a
 x + y + z 
 97. What is the remainder of  c 
three digit number.  b 
(1) x + y + z = 25 (1) a, b, c are all even numbers.
(2) Unit digit of x y is same as that of y z (2) b is a prime number while a and c are
 xy  composite numbers.
91. If remainder of 
 7 
 is 1 then what is the
98. What is the remainder of 
{
 abc abc 
}
 here ‘abc’
 x  xa + yb + c 
remainder of   ?
7 is a three digit number and x, y are +ve integers.
(1) Given that y is a two digit prime number. (1) x is 10 times of y
(2) Given that y is four digit composite number (2) y is smallest two digit number that has 4
not divisible by any digit except 1. factors.

92. What is the remainder of 


 x97531 
 24 
 
99. What is the remainder of 
{ }
 ( n!) 2 

 2 n + 1 
 x (1) 2n + 1 is a prime number and n is odd
(1) Remainder of   is 4
6 (2) 2n + 1 is a prime number and n is Even
 x 100. Find the unit digit of a! + b! + c!
(2) Remainder of   is 3
4 (1) Each of a, b and c is a two digit number.
 xy  (2) Each of a, b and c is a prime number in the
93. What is the remainder of   form of 6k ± 1.
 36 
x  x  a !
(1) Remainder of   is 1 and that of   is 1 101. Find the remainder of  
18  8  b! 
(1) Given that a ≥ b
x  x
(2) Remainder of   is 17 and that of   is (2) Given that a ≤ b
18  8
7 102. Find the highest power of 7 in N!.
 x ! N
94. Find the remainder of   (i) Quotient of   is 7
y 9
(1) Given that |x – y| =2 N
(2) y is a three digit prime number and more than (ii) Quotient of   is 5
11
x. 103. Find the last two digits of a! + (a + 1)! + (a + 2)!


95. Find the remainder of 
{ }
 ( x!) 2 

(1) a is a prime number.
(2) a is a two digit number.
 2x + 1 
104. Is a! divisible by 72?
(1) Given that x is Even.
(1) a! is divisible by 18
(2) Given that x is Odd
(2) a is divisible by 4 but not by 8, a is divisible
 ( aa )bb 
96. What is the remainder of   here a, and b by 3 but not by 9.
 100  105. Is a! divisible by 68?
are digits
 ab  (1) a! is divisible by 17.
(1) Remainder of   is 29 (2) a! is divisible 26.
100 
106. What is highest power of 3 in a! ?
 ab  (1) Highest power of 9 in a! is 5
(2) Remainder of   is 56
100  (2) Highest power of 9 in a! is 6
Buy books : http://www.dishapublication.com/entrance-exams-books/mba-exams.html
16
107. What is highest power of 9 in a! ? 118. What is number of zeros when N! is converted in
(1) Highest power of 27 in a! is 12 hexadecimal system (Base 16)
(2) Highest power of 27 in a! is 9 (i) Number of zeros at the end is 8 when N! is
108. Find the unit digit of (N!) converted to base 9.
(ii) Number of zeros at the end is 8 when N! is
(1) Unit digit of (N-5)! Is not 0.
converted to base 12.
(2) N is divisible by 3 but not by 6. 119. What is last two non zero digits of nCr
109. Find the number of zeros at the end of N! (1) n = r + 2
(1) Highest power of 2 in N! is 97 (2) Highest power of 2 and 3 in n! is 11 and 6
(2) Highest power of 4 in N! is23 respectively.
110. Find the number of zeros at the end of N! 120. Is A > 1 ?
(1) Highest power of 10 in (N+1)! Is 12 (1) Log A X > Log A X2 ( X is an integer)
(2) Highest power of 10 in (N-1)! Is 10 (2) A2 < A
111. What is HCF of a! and b! ? 121. What is the value of (a + b)3 ?
2
(1) a is root of quadratic equation x – 5x – 6 = 0 (1) a2 + b 2 = 13
(2) b is a two digit even number while ‘a’ is a (2) a*b = 6
single digit even number. 122. What is the value of X?
(1) if log 2, log (2x -1 ) and log ( 2x + 3 ) are in
112. Find the number of zeros at the end of (a!)(b!)
AP
(1) Highest power of 2 in a! is 41 and highest
(2) if X is real number
power of 5 in b! is 12
123. Is X > Y ?
(2) Highest power of 2 in a! is 46 and highest (1) X2 + X = 2 and Y2 + 2Y = 15
power of 5 in b! is 24
(2) X2 – 2 X = 8 and Y2 – 3Y = 18
113. Find the number of zeros at the end of (a! + b!) 124. If X and Y are positive integers then is ( X2 + Y2 )
(1) Highest power of 2 in a! is 41 and highest
greater than  +  ?
1 1
power of 5 in b! is 6  x y
(2) Highest power of 2 in a! is 46 and highest (1) X > Y
power of 5 in b! is 24 (2) XYZ = 1
114. Find the unit digit of (a!)(b!) 125. X2 – 32X + 216 = 0 what is the value of X?
(1) a = b + 7 (1) X is perfect cube
(2) b = a + 4 (2) X2 is perfect cube
115. What is the last two digits of [a!/b!] if a > b ? 126. What is the value of x?
(1) log ( X2 + 10X + 25 ) = 2
(1) Given that a = b + 7 and b is not a multiple of 5
(2) log ( X3 – X2) = 2
(2) Given that a = b + 4 and b is a multiple of 5
127. Is X > Y?
116. What is the largest prime number that has exponent (1) X /Y = 8/7
2 in N!. (2) X2 > Y2
(i) 17 is the largest prime number that has 128. What is the value of C in this equation X2 + X +
exponent 2 in (N/2)!. C=0
(ii) 19 is the largest prime number that has (1) the equation has two roots
exponent 2 in (N/3)!. (2) the sum of the roots is -1
117. What is the highest power of 3293 in N! 129. Is X3 + Y3 + Z3 = 3XYZ ?
(ii) Highest power of 17 in (N/2)! is 3. (1) X +Y+Z = 0
(i) Highest power of 37 in (N/2)! is 2. (2) X2 + Y2 + Z2 = XY +YZ + ZX
Buy books : http://www.dishapublication.com/entrance-exams-books/mba-exams.html
17

HINTS & SOLUTIONS


Concept Applicator Now if we consider Statement I alone
Product of these 7 integers = 702800
1. (d) If we look at Statement I then we will get
Since 702800 = 24  52  (251)(7), it cannot be
If a = 3 and b = 2, a + b > 0. Here b > 0
the product of 7 consecutive whole numbers.
If a = 3 and b = –2, a + b > 0. Here b < 0
Hence I alone is insufficient.
Hence I alone is not sufficient. Now if we consider Statement II alone
Now if we look at Statement II only then we Given that their sum = 105 = 7n or n = 15 and
will get then 7 consecutive integers are 12, 13, 14, 15,
If a = 3 and b = 2, a – b > 0. Here b > 0 16, 17, 18
If a = 3 and b = –2, a – b > 0. Here b < 0 So, II alone is sufficient.
Hence II alone is not sufficient. 5. (a) Since sum is 360 hence P + Q + R + S = 360
Now by using statements Ι and ΙΙ together From statement I alone we will get P =
If a = 3 and b = 2, a – b > 0 and a + b > 0. Here (Q + R + S)/3 from this we can find the value of
b>0 P hence statement I alone is sufficient enough.
If a = 3 and b = –2, a – b > 0. and a + b > 0. From statement II alone we can not find the
Here b < 0. value of P.
Hence Ι and ΙΙ together are also insufficient.  6. (d) Statement  Ι  is not sufficient as the size of
2. (c) If we look at Statement I the ice cube and height of the container is
i = p – 17 and r = p – 103 not known hence statement I is not sufficient
Hence, we cannot find how many each alone.
received so this statement is not sufficient Statement  ΙΙ  is also not sufficient as the
enough. dimension of the container is not known.
We cannot answer the question even by
Now by considering Statement II alone.
combining both the statements as the height
p + i + r = 170
of the container is not known.
Hence, we cannot find how many each 7. (b) It is given that Ram got a dividend of ` 1500.
received. so this statement is not sufficient Statement I
enough Knowing the dividend paid last year, we
Using I and II together, we get p + (p – 17) + cannot find the dividend paid this year.
(p – 103) = 170. Statement ΙΙ
Solving the above equation we get the value Given that Ram bought 350 shares of face
of p and the values of q and r. value  ` 10, and so, their total face value is
3. (b) If we look at Statement I ` 3500. So here we know the investment and
It is given that the circles are concentric. the return hence we can find out the rate of
But nothing is given about their dimensions. interest.
Hence Ι alone is not sufficient. 8. (d) Consider Statement I alone.
In statement II ratio of area is given hence we Given that Area (∆ ABC) = Area(∆PQR)
can find the required ratio. since nothing about the sides or angles
4. (b) Let the 7 consecutive whole numbers be is mentioned, we cannot say if they are
(n ± 3), (n ± 2) (n ± 1), n. congruentHence, Ι alone is not sufficient.

Buy books : http://www.dishapublication.com/entrance-exams-books/mba-exams.html


18
Consider Statement ΙΙ alone Consider statement I alone:-
∆ABC and ∆PQR are right triangles.Nothing we know that Ram is not the shortest, either
about the sides is given, hence, ΙΙ alone is not Shyam or Jay is the shortest.
sufficient. Hence (Ι) alone is not sufficient.
Now using both Ι and ΙI Consider statement I aloneShyam > Vikram.
Now we have two right angled triangle with From the given information and the information
same area we may have different combination in (ΙΙ), we get Ram > Shyam > Vikram > Jay.
as only product of base and height is same. Hence, (ΙΙ) alone is sufficient.  
Hence even by using both the statement we
can not find the answer. Concept Builder
9. (a) Given that their salaries are in the ratio of
1. (a) Statement Ι alone is sufficient.
3 : 4 and expenditure is in the ratio of 4 : 5
Statement ΙΙ alone is not sufficient, for we can
hence we can assume that salary of A and B
have more than one value of MN possible.
are 3x and 4x and their expenditures are 4y
and 5y. 2. (e) Given relationship is (PQ)(RQ) = XXX
Now we need to find the ratio of (3x – 4y)/ Since X can take 9 values from 1 to 9 hence
(4x – 5y) we have 9 possibilities
Consider statement I alone:- 111 = 3 × 37  444 = 12 × 37  777 = 21 × 37
Saving of B is 25% of his salary hence his 222 = 6 × 37  555 = 15 × 37  888 = 24 × 37
expenditure must be 75% so ¾(4x) = 5y or 333 = 9 × 37  666 = 18 × 37  999 = 27 × 37
3x = 5y from this we can find the required But out of these 9 cases only in 999, we get
ratio hence this statement is sufficient. the unit’s digit of the two numbers the same.
Consider statement II alone:- Since it is a unique value, hence we need
Given that 4x = 2000 or x =500 but from this neither statement Ι nor statement ΙΙ to answer
we can not find the value of y and hence we the question.
can not find the ratio of their savings. 3. (d) C
10. (c) Let x be the average height of the class and n
be the number of students in the class.
Consider statement I alone:- a cms
b cms
xn –  56 = (x – 1)(n – 1)
⇒ x + n = 57 …(1)
Hence, the value of x cannot be found. 60°
So, Ι alone is not sufficient. A B
Consider statement II alone:- 60 cms
xn –  42 = (x + 1)(n – 1) Let, a cm and b cm are the the two unknown
⇒ x – n = 41 …(2) sides as shown in the fig.
Hence, the value of x cannot be found. From statement 1, a + b = 80 cm, hence
So, ΙΙ alone is not sufficient. b = (80 – a) cm
Both the statements together are sufficient as Now using cosine rule.
the value of x can be found by solving (1) and (2) Cos 60° = ( AB2 + AC2 – CB2 )/2 AB
11. (b) Given that Ram > Shyam, Vikram > Jay. 1
∴ = [602 + b2 – (80 – b)2]/120
Hence from this we can conclude that neither 2
Ram nor Vikram is the shortest. And we have By solving this we get, b = 28 cm. Hence,
to find the shortest among them - statement 1 is sufficient to answer.
Buy books : http://www.dishapublication.com/entrance-exams-books/mba-exams.html
19
From statement 2, If we combine the two statements; only case
Since ∠B = 45° hence ∠C = 75° (iii) is possible.
According to sine rule: we know that a/sin a Hence (c) is correct option.
= b/sin b = c/sin c = a/sin45° = b/sin60° 8. (e) From statement 1: F(x = 10a + b) = f (a) + 12b
= 60/ sin75° = 1485, from this equation we will have
From this we can find the value of the sides. different values of a, and b and hence many
Hence statement 2 is sufficient to answer the values of x. From statement 2: Maximum
question. value of b = 9, since we don’t know the exact
4. (a) From statement I, value of b so we cannot determine. From both
E + B < A + D, we easily say that E is less than equation we cannot solve due to unknown
A, because B > D and as the statement suggest value of f (a).
E + B < A + D.  ∴  E < A.
∴  A is not the smallest integer. Concept Cracker
Statement I is sufficient to answer. 1. (d) Statement A: Alone – it gives highest and
From statement II, D < F lowest weight of members of the class. This
This statement is not sufficient to find the does not give any other information so we
relation between A and E. cannot determine the number of students.
5. (e) Since ‘b’ is the common difference of three Hence this statement is not sufficient enough.
sides of a triangle, a/b can take any values in Statement B: Alone – It gives that by
the given range of real; numbers.
eliminating the highest and lowest members,
Suppose a = 1000
b=1 the average weight does not change. This is
Then there sides will be 1000, 1001, 1002 also not sufficient to answer anything.
Here a/b = 1000/1 = 1000 Both the statements together cannot determine
Hence the values of ‘a’ and ‘b’ can be the number of students as the average weight
varied to any values making the ratio a/b of students (after eliminating highest and
undeterminable. lowest) cannot determine the total number.
6. (c) From statement 1: Let AB=2x and BC=2y, 2. (c) To find the volume, we need radius of sphere.
then AC2 = 4(x2 + y2), since we don’t know the Statement A alone - does not give any useful
values of x and y hence we cannot calculate information.
the AC, hence this statement is not sufficient Statement B alone – it says 20 litres of water
enough. are displaced when the tank is fully immersed.
Statement 2: AD=7 and BD=5; We don’t get Hence we will get the outer volume of the
answer from this spherical tank. Now with this the outer radius
Now combining both the information we can of the tank can be found. But this is not
write (2x + y)2 = 49 and (2y + x)2 =25, hence sufficient to get an answer.
we can calculate the value of x and y and On combining we can find the inner radius
hence we can calculate the value of AC because the wall is 1cm thick. And hence the
7. (b) We have three possible cases
storage volume can be determined.
Case (i) DBEAC
3. (c) Statement A alone – It is given here that the
Case (ii) BADCE
Case (iii) CADBE shape of the field is rectangular but exact
From statement (1), either case (ii) or case location of point A, B and C is not given hence
(iii) could be valid hence we cannot get a we cannot find the from this information.
unique value from this statement alone. Statement B alone – It is given that the shape
From statement (2), either case (i) or case (ii) is polygon but we don’t know whether it is
is possible hence we cannot get a unique value polygon of n = 4, 5, 6 or anything else, hence
from this statement alone. this is not sufficient to get an answer.
Buy books : http://www.dishapublication.com/entrance-exams-books/mba-exams.html
20
By combining statement A and B we know 8. (c) Statement A states that A and B passed.
that shape is rectangle and C and B are its Statement B actually means neither C nor D
vertices and A is the mid point of other side, has passed.
hence together both the statement is sufficient Hence from both the statements we can
to get answer. determine that the total number of students
4. (a) Here we need to find the ratio of growth and who passed was 2 and these are A and B.
that will be equal to the ratio of the slope of 9. (c) Statement A alone – it gives the following
the line. Statement A- From this statement equation Log2 X = √X X = 2√X. This
we know the slope and hence we can find out is not sufficient to answer the question.
the ratio of slopes. So this is sufficient to get Because there are many values of X
answer. Statement B- From the scale only we may satisfy the equation, e.g., 4, 16 etc.
can not calculate the ratio.
Statement B alone – it says X ≤ 10. This is
5. (a) Statement A- alone- With the coordinates of
not sufficient by itself to answer the question.
point D given it helps to find out the equation
But on combining both the statement A and B
of the straight line passing from the two points
we get only the integer 4 that shall satisfy the
where the tangents meet the circle, i.e., (r, 0)
equation.
and (0, r). And if we know the equation of a
straight line then we can calculate its slope and 10. (d) Since Black Seed gives Blue flower or Red
angle between tangent and x-axis and hence flower. White seed gives White flower or red
this statement is sufficient enough. Statement flower. Statement A alone – it says plants with
B does not give any answer. So answer is (1) white flowers are 10. Hence this statement is
6. (d) Statement A gives (kd)x + (ke)y = k f, k ≠ 0 not sufficient enough to answer the question.
⇒ dx + ey = f ⇒ These represent one and  the Statement B alone – it says number of plants
same equations, hence infinite pairs can satisfy. with red flowers is 70. Again nothing can be
hence this is not sufficient to get an answer. concluded. On combining we get 10 plants
Statement B gives x + y = c, 2x + 2y = f with white flowers and 70 with Red flowers.
⇒ f = 2c. but it is given that f ≠ 2c ⇒ So plants with Blue flowersshould be 20 (As
Inconsistent. Combining the two statement A total number of plants is 100). Now blue
and B also we cannot determine the answer. flowers shall come from Black seeds only. So
7. (b) Statement A alone- it says a mathematician there are at least 20 Black seeds used. 10 white
can never add two numbers correctly, but can flowers can come only from white seeds. So at
always add three numbers correctly. Let there least 10 white seeds have been used. Now the
are three professor A, B and C, remaining 70 red could have come from either
A gives an answer 1 + 1 = 3, hence he is a black or white seeds. So data of this statement
mathematician. is insufficient.
B gives an answer 1 + 1 + 2 = 3, hence he is Even by using both the statement we cannot
not a mathematician. determine the answer.
C gives an answer 1 + 1 = 3, hence he is a 11. (c) A alone : X may be greater than Z. But doesn’t
mathematician.
say anything about Z being smallest.
So this statement is sufficient enough
B alone : Y may be greater than Z. But says
Statement B- says when a mathematician
makes a mistake in a sum, the error is +1 or nothing about Z being smallest
–1. Here two professors madea mistake of +1 Now we will use both statement A and B
and –1, the third got it correctly. Hence this If X > Y, then  Y > Z and   X > Y > Z
statement is sufficient enough Hence each But if X > Z, then Y > X and  Y > X > Z
statement individually is able to answer the In either of these two cases, Z is the least and
question. hence the question can be answered.    
Buy books : http://www.dishapublication.com/entrance-exams-books/mba-exams.html
21
12. (a) From A alone: x(x + 3) < 0 hence  0 > x >  – 3 Statement A: Gives us the radius of
Hence, |x| is necessarily less than 3 in this inscribed circle = 5,but with this information
range. we cannot calculate PR & QR.
From B alone : x (x – 3) hence x > 3 or x < 0 Statement B : Gives hypotenuse PQ = 18 cm
In this condition, any given |x| value need (dia of circumcircle).
not necessarily be less than 3. So B is not
Now, ∠ P + ∠ Q = 180° – 90° = 90°
sufficient enough.
13. (c) Consider A alone : from the formulan ( P U Q) So ∠P/2 + ∠Q/2 = 45° but this is not sufficient
= n(P) + n(Q) – n(P ∩ Q) enough.
We get n (P U Q) = n (P) + 1000 – 100. Now, If IS = 5cm (in radius = 5), then
From A alone, we get 2 unknowns and 1 Using both the information we can calculate
equation.  Hence  A alone is not insufficient 15. (a) Consider A alone : Given S.P / C.P = 1.05 and
enough. brokerage of 1%
Consider B alone: n(P U Q) = 1500 given, but
from Balone, n(P) cannot be determined. Let CP =100x brokerage = x so total CP = 101x,
Consider both together :1500 = n(P) + 1000 – and SP = 105x
100 AND n(P) = 1500 – 900 = 600 A is sufficient.
Hence both the statement required to solve Consider B alone : No. of shares = 100, since
the question. no price is given, profit cannot be calculated
Alternately- From statement A alone hence this statement is not sufficient enough.
P Q = 1000 16. (c) From the main information a@b is not define
when a or b is zero.
a c Consider A alone : from this information we
b
see that
a @ b   = 0 only if a is zero.
However in the expression (2 + 0) : a ≠ 0. A is
It is given that b + c = 1000 and b = 100 hence not sufficient.
c = 900 but we cannot calculate a, so this B alone is not sufficient enough as it does not
statement is not sufficient enough. provide important information.
From statement B alone a + b + c = 1500, Consider A and B together a @ b = b @ a
and this statement is also not sufficient enough hence 2 @ 0 = 0 @ 2
to find the value of a + b
If we use both the statement then we can So (2 @ 0 ) = 0 and (– 5 @  – 6) =1
calculate the value of a + b Hence A and B together is sufficient enough
14. (c) P 17. (d) Two straight line intersect if they are not
parallel hence if a/d ≠ b/e
R Now consider statement A alone, it does not
S say specifically that a/d ≠ b/e
From statement B alone, if c and f both are
zero then also we cant say that a/d ≠ b/e

I Even with both the statement together we
 cant determine whether the two straight line
R Q
intersect or not.
Buy books : http://www.dishapublication.com/entrance-exams-books/mba-exams.html
22
18. (b) From statement A 21. (c) Statement A- it is not sufficient to give the
answer as we will get the pairs (15,2), (10,3),
(6,5). There are infinitely many values.
But using the statement B we can say the that
value of (m, n ) may be, (1,30), (2, 15), (3, 10),
O
B D (5, 6) hence this is not sufficient enough.
A E
C By combining A and B we will get unique
answer m = 15, n = 2.
22. (d) Statement A- as we don’t know that what
We see that BC = CD or C is at the center of were their GDPs 5 years ago. Hence we cant
B and D    compare its present value.
So C will also be the centre of A and E, Hence Statement B- we know that GDP of country X
(A) is sufficient to answer. is greater than that of Y but we do not know
From Statement (B) by how much. Hence even using both the
statement we cant determine.
23. (a) Considering statement A alone the possible
pairs are (5,1), (7,1), (9,1), (2, 6), (9, 3). So there
B
OC
D is only one pair with both even numbers (2, 6).
A E Now considering statement B the possible
T pairs are (2, 6) and (6, 2). So there is no unique
solution by using (B) alone. We can get direct
answer from statement A alone.
Let T be the center of the third circle. Then
24. (a) From statement A- We cannot make out the
since C is the line joining O and T implies C is number of rounds, as maximum capacity is
midpoint of B and D or bisects B and D. given 200, but actual value is not given.
Similar to last case now we can say that this
Statement B- However, we can find out the
statement is also sufficient enough.
trip time using the given values and can thus
19. (d) Consider statement A alone : Avg. speed calculate the total number of trips since total
of plane says nothing about local time in no- time is fixed 12 hrs.
mans land. Statement B alone : Distance also
25. (c) Statement A: alone it is not sufficient enough
says nothing about the local time.
because it gives only the data transfer rate but
Now if we use both the statement together
does not give the size of the software to be
: Time required for flight can be obtained but
downloaded.
still we cannot find the local time.
Statement B: alone it is also not sufficient
20. (d) From statement A alone it is given that x – y
because it gives only the size of software but
= 6 or y – x = 6, from this we cannot find out a
does not give the rate of data transfer.
unique value of x or y, hence this statement is
not sufficient enough. Combining both the statements, we can get
From statement B xy = 6k, again we cannot the answer.
26. (b) We know that if diameter is given, then side
find out a unique value of x or y, hence this
statement is also not sufficient enough. of square can be found. Similarly if side of the
Now if we use both the statement together square is given then the radius of circle can be
then, let x – y = 6 and xy = 6k, or x = 6k/y or found.

6k/y – y 6 or y2 + 6y = 6k, and again we will Hence, both the statements individually can
not get a unique value of x and/or y. provide the difference in areas.
Buy books : http://www.dishapublication.com/entrance-exams-books/mba-exams.html
23
27. (d) Ratio of number of apples bought by Ram A tells us F has 2 brother then they must be
and Gopal is given and total quantity at dealer unmarried. Rest two uncle must be brother of
is given, but what part of the total quantity M and married. Rest aunt of s should be M `s
they bought is not given hence even both sibling.
statements are not sufficient to get the number We get answer from A.
of apples bought by Ram and Gopal. From B is not correct to get answer.
28. (d) We do not know whether Korea scored a goal 37. (b) Let the total no. of time coin tosses be n.
in the last 5 minutes or it may be possible that Total amount spent by ram = (10 + n × 1 )
india at begining has 0 goal and korea has 2, = 10 + n
so at the end they may draw, even from both First he paid ` 100 but he suffered ` 50 losses.
statement we can not conclude. Then amount left with him,
29. (a) From Statement A it gives us that by adding (10 + n) – 100 = 50 ⇒ n =140
4, the number (4, 12, 20…) would be divisible We get result using A.
by 8. Hence it is sufficient enough For statement B we have 2 cases
30. (a) From statement A - we get (x + y)(x + y/xy) = Case (1): Game ended normally – this case is
2
4; or (x + y) = 4xy ; similar to statement A
2
Or ( x – y ) = 0, hence x = y. Case (2): Game ended prematurely – this case
From statement B- (x – 50)2 = (y – 50)2, is not possible with 138 tails.
or x – 50 = +/– (y – 50) Hence both the statements are sufficient
Hence we will get x = y or x + y =100 hence enough
We cannot get the answer from the second 38. (c) Price of each soap= ` 10
statement. From statement 1 : She spent total ` 210.
31. (a) Statement A – After reducing the listed price So, she bought 210/10=21 soaps. And get last
by 10 percent, the dress sold for a net profit letter S. We cannot say anything from it.
of 10 dollars and given that net profit is 20% From statement 2: Total vowels = 18 means
hence we can get the CP. he got no. of A and O=18
But statement B just gives the SP but we do From it we cannot say anything
not know the discount. But using both, no. of P she got = 21– (18 + 1)
32. (d) We cannot arrive at the even with the use of =2
both the statement. Average since we do not 39. (c) Race of A and C, A wins by 375 m and A wins
know individual scores or number of students. by (60 + 30) = 90s
33. (d) After putting different values in the given Speed of c = (375/90) m/s. Time cant be
statements. We find that the question cannot calculated because we don’t know the length
be answered even by the use of both the of track.
statement as we get different answers. From statement B, length of track = 1 Km.
34. (c) If we draw a Venn diagram we will find that, from it nothing is concluded.
we need both statements. But using A and B we get speed of C = 1000/
Hence by using both the statement we can (375/90) .
solve it. 40. (a) Assume A, B, C, D get score 10, 8, 6, 4
35. (c) From statement A we get only J’s share. ABCD
Statement B also alone can not solve it 10 8 6 4
But by combining the statements we get the Statement A:
values of each student. With the conditions A will give vote to B
36. (a) From above question, Father and Mother of With the conditions B will give vote to A
S is F and respectively. Total no. of uncle and With the conditions C will give vote to A
aunts are 4 and 3 respectively. Even if D gives to A/B/C–2 situation arises.
Buy books : http://www.dishapublication.com/entrance-exams-books/mba-exams.html
24
Either A will win or there will a tie when D 46. (d) Let the abbreviation of their name as A,B,C,D
gives vote to B. and E,
Even then A will win. From main information D>E, B >C, and the
So we are getting the answer. rank of C is either 4th or 5th in descending
Statement B: Cannot conclude anything. order. And we need to find the highest ranker.
41. (a) We have to find out, which one greater: 30% Now From Statement 1- it is given that A
of Z or 40% of S. ranked 5 and any one of the B or D can be
Statement A: 20% of Z > 25% of S; So, we the top ranker, this statement is not sufficient
can not conclude anything about 30% of z : enough as any one of B or D can be top ranker.
40% of s; Statement 2- B’s rank is either 3, 4 or 5, again
Statement B: 13 % of S > 10% of Z => 39 % we don’t have clear information that who is
of S > 30 % of Z the topper.
Which further implies, 40% of S > 30 % of Z Combining the statement 1 & 2 we can
42. (b) Statement A: We can find, there are 12 Tails conclude that D is the top ranker as from
and 9 Heads. statement 2 B cannot be the topper.
After tosses he will reach at blue point. So 47. (c) Let the number of employee be 100x
statement A is sufficient. Then the number of female employee is 70x,
Statement B: 3 more Tails greater than and those with engineering background 7x
Heads. So he will reach at blue point after Statement I- the no of employee with
tosses. So statement B is also sufficient. engineering background is 25x
43. (a) Statement A: Cannot say anything. The % of male employees with engineering
Statement B: Because amongst the Top 5 → background out of the males = (25x-7x)/30x
3 are boys, 2 are girls. And Rashmi is third ×100
among the girls and Kumar is 6th. Similarly the % of employee out of total
So, B is sufficient. employees can be calculated.
44. (d) Statement A: 13 currency notes will give Hence this statement is sufficient enough.
different possible Values and hence it is not Statement II- the number of male employees
sufficient. Statement B: Multiple of 10 and having engineering background is 84x
will give many values. Similar to above calculation this statement is
Even if you combine the statement, we can also sufficient enough.
have various values. 48. (e) Statement I- this statement doesn’t give
45. (c) Statement A: 2 kg potato cost + 1 kg gourd information about the number of goals scored
cost < 1 kg potato cost + 1 kg gourd cost or by the opponent team, hence this statement is
1 kg potato cost < 1 kg gourd cost. Hence not sufficient enough.
statement A is not sufficient. Statement II- This statement doesn’t give
Statement B: 1 kg potato cost + 2 kg onion information about the number of goals scored
cost = 1 kg onion cost + 2 kg gourd cost by Mahindra & Mahindra club, hence this
Hence 1 kg potato cost + 1 kg onion cost = 2 kg statement is not sufficient enough.
gourd cost. Combining both the statement even it is not
So statement B is also not sufficient as we can given the number of goals scored by Mahindra
not conclude. & Mahindra in 1sthalf, hence even by using
Combining both statements we get 1 kg potato both the information we cannot find the result.
cost < 1 kg gourd cost (From statement A)…(i) 49. (a) From statement I- 40% of total is given 10
1 kg potato cost + 1 kg onion cost = 2 kg hence total is 10/0.4 = 25.
gourd cost (From statement B) …(ii) This statement is sufficient enough.Statement
So we can conclude that the onion is costliest. II- It does not give any significant information.
Buy books : http://www.dishapublication.com/entrance-exams-books/mba-exams.html
25
50. (d) Statement A alone: Since we do not know the 3. (e) From statement (1) we cannot conclude
number of byes got by the champion. Hence, anything.
statement A alone is not sufficient to answer
From statement (2) since N is a perfect square
the question. as well as perfect cube hence N = P6 so
Statement B alone : Since we do not know possible values of N = 16 = 1, or 26 = 64, or
the exact number of players in the tournament. 36 = 729.
Hence, statement B alone is not sufficient to
From both the statement we can have N = 1 or
answer the question. 729, hence we can find exact value of N
Combining both the statements together:
4. (a) From statement (1) alone since sum of two
If there are 83 players, then there will be 6
terms is even hence either both of them are
rounds in the tournament and we know that
even or both of them are.
the champion received only one bye, therefore
the total number of matches played by the Case (i) when both the terms are even then
tournament will be 6 – 1 = 5. Given in option 4 abc = even and def = even and hence abc + def
51. (d) Statement A alone : When n = 127, exactly is even
one bye is given in round 1. When = 96, Case (ii) when both the terms are odd then
exactly one bye is given in round 6. As no abc = odd and def = odd and hence abc + def
unique value of n can be determined hence, is even
statement A alone is not sufficient. Hence from this statement we can say that
Statement B Alone: Since we do not know abc + def is even.
exactly how many bye’s are given, so, we From statement (2) alone again we have two
cannot determine the value of n, uniquely. cases
Combining statement A and B: using both
Case (i) when both a and d are even then both
the statement. There is a unique value of n =
abc and def are even hence abc + def = even
120, for which exactly 1 bye is given from the
third round to the fourth round. Case (ii) when both a and d are odd then we
cannot say anything about ‘abc’ or ‘def’ as we
Concept Deviator don’t have information about b, c, e and f.
Hence this statement alone is not sufficient
1. (c) From statement (1) alone we cannot say
enough.
anything as, K = 1/3 (When P = 3) or K can be
integer. 5. (e) From statement (1) alone we can’t answer
From Statement (2) alone we can’t deduce the question.
anything as nothing is given about P From statement (2) alone again we can’t
From both the statement if P is prime and K/P conclude anything.
is integer then K must be an integer. 6. (e) From statement (1) consider P = 2 + √3 and Q
2. (c) From statement (1) alone we know that A is = 2 – √3 then P + Q is rational but if P = √2
an even number and B is prime from this we and Q = √3 then P + Q is irrational hence this
cannot conclude about A/B. statement is not sufficient enough.
From statement (2) alone consider A = 2 and From statement (2) consider P = 2+√3 and Q
B = 4 then A/B is not an integer but if A = 4, = 2 – √3 then PQ and P + Q both rational but
B = 4 then A/B is an integer hence this if P = √2 and Q =2 √2 then PQ is rational but
statement is not sufficient enough. not P + Q hence this statement is not sufficient
If we combine both the statement then only enough.
possibility is A =4 and B =2 or A = B = Prime Even we use both statements then also we
number in both cases A/B is an integer. will have same situation.
Buy books : http://www.dishapublication.com/entrance-exams-books/mba-exams.html
26
7. (c) From the given information 0. abcd − ab
−abc − ab 100a + 10b + c − 100a − b P 12. (d) Since a.bcd =

= abc = = 900
900 900 Q
{(1000a + 100b + 10c + d ) − (10a + b )}
Hence P + Q = 90a + 9b + c + 900 is divisible =
900
by 9 only when c = 0 or 9
From statement (1) alone a + b + c is divisible = 990a + 99b + 10c + d = P/Q
by 9 from this we can not conclude about c 900
whether it is 0 or 9. Hence P – Q = 990a + 99b +10c + d – 900
From statement (2) alone a + b, b + c, and is divisible by 9 that means 10c + d should
c + a are divisible by 9 but from this we can be divisible by 9 or two digit number ‘cd’ is
not conclude about c whether it is 0 or 9. divisible by 9 or c + d divisible by 9.
From statement (1) two digit number ‘dc’ is
If we combine both the statements then since
divisible by 9 hence c + d divisible by 9 hence
a + b and a + b + c both divisile by 9 hence c
this statement is sufficient enough to find the
must be divisible by 9.
answer.
8. (c) From statement (1) alone we can conclude From statement (2) alone when two digit
that at least one of a or b must be positive number “dc” is divisible by 6 then resultant is
hence a^b + b^a may be positive or negative. not divisible by 3 hence “dc” is not divisible
From statement (2) alone either both of them by 9 so this statement is sufficient enough.
are positive or both of them are negative, 13. (d) Here in this case either statement is sufficient
hence a^b + b ^a may be positive or negative. enough to answer the question.
If we use both statements then we will get From statement (1) and (2) alone we are sure
both a and b are positive. that the number is prime
9. (b) From statement (1) alone we can say that the 14. (c) From statement (1) we do not have any
number is divisible by 2, 3, 5 and 7 i.e. 210, information about B and similarly statement
from this we can’t conclude that “abcd” is (2) alone is also not sufficient enough.
divisible by 4 or not. From statement (1) and (2) both we are sure
that even if A + B cannot be rational.
From statement (2) alone the number is
15. (e) None of the statement says that a, b and c are
divisible by (n –1)n(n +1) where n is odd hence
integers hence even with both the statements
the number is divisible by two consecutive
we cannot answer.
even numbers hence it must be divisible by 4.
16. (e) From statement (1) and (2) alone no
10. (e) From both the statements (1) and (2) it is not information is given about third number it
clear whether a, b and c are integers hence may be integer or it may be fraction.
we can’t conclude even by using both the If we combine both the statements then we
statements together. have 2 even integers and we may have 1 or
11. (a) From statement (1) alone digits are 1, 2 and none even negative integers, hence even by
3. Since sum of digit is 6 hence it can’t be using both the statements we cannot answer
divisible by 9 hence this statement alone is the question.
sufficient enough. 17. (a) Here we have to find the minimum value of
From statement (2) alone the number is A/B + B/A let us assume that A/B = t then
divisible by 210 hence possible values of N is A/B + B/A = t +1/t, we know that its minimum
210, 420, 630 and 840, out of these only 630 value is 2 if t is positive i.e. A/B is positive
is divisible by 9 hence we cannot conclude i.e. either both of them are positive or both of
from this statement. them are negative.
Buy books : http://www.dishapublication.com/entrance-exams-books/mba-exams.html
27
From statement (1) alone since AB > 0 hence 22. (c) From statement (1) alone:- If a number has 12
either both of them are positive or both of factors then possible ways of writing N is a11,
them are negative hence this statement alone a1b5, a2b3, a1b1c2
is sufficient enough to find the answer. Then N2 must be one of a22, a2b10, a4b6,
From statement (2) alone at least one of them a2b2c4 and corresponding number of factors
is positive from this we cannot be sure about of N2 is 23, 33, 35, 45, hence statement (1)
the parity of A/B. alone is not sufficient enough.
18. (e) From statement (1) alone we can’t determine From statement (2) alone number N has three
as third one may be positive or negative. prime factors but from this we can not say
Similarly from the 2nd statement. anything about N.
If we use both the statement then we will If we combine both the statements then
get one negative and two positive but we number must be in the form of a1b1c2 or
N 2 = a2b2c4 has 3 × 3 × 5 has 45 factors.
don’t know magnitude of these numbers i.e.
(negative is how much is it –1 or –1000) 23. (e) From statement (1) alone A is prime hence it
hence we can’t answer even with both the has only 2 factors and B and C must be either
statements. 4 or 9 (3 factors)
19. (c) From statement (1) alone: - since 46 + 47 + From statement (2) alone we can not find any
st
48 + 49 + 50 = 240 hence 1 of these natural conclusion.
numbers must be more than 45. If we combine both the statements then we
Similarly from statement (2) alone: - since 50 will find that B = 9 and C = 4 but both has
+ 51 + 52 + 53 + 54 = 260 hence smallest of same number of factors, hence even by both
these numbers must be less than 50. the statements we can not find out the result.
From both the statements together we are 24. (b) From statement (1) alone we have two prime
sure that one of the number must be 50 hence numbers 79 and 97 hence this statement
product must be divisible by 25. alone is not sufficient enough to answer the
20. (d) From statement (1) alone question.
235, 253, 237, 273, 257, 275, From statement (2) alone we have 67 and 76
325, 352, 327, 372, 375, 357 but out of these two only 67 is prime hence
523, 532, 527, 572, 537, 573 this statement alone is sufficient enough.
723, 732, 725, 752, 735, 753 25. (d) Odd number of factors means perfect square.
Hence we have 24 such numbers and 4 such If N/6 has odd number of factors then N/6 =
sets K2 or N = 6K2
Similarly from statement (2) alone we can From statement (1) alone N/24 = P2 or N =
24P 2 = 6(2P)2 hence N/6 = (2P)2 a perfect
find out numbers of such sets.
21. (c) Odd number of factors means number should square hence has odd number of factors, so
be perfect square this statement alone is sufficient enough.
From statement (1) alone we can’t sure about From statement (2) alone 24N = Q2 or N =
the number is perfect square or not. Q2/24 hence N/6 = (Q/12)2 is a perfect square.
From statement (2) alone number N must be Hence this statement alone is sufficient
one of 62, 63, 64, 65, or 66, out of these only enough.
64 is a perfect square. 26. (d) If N has 81 factors then N must be in one
If we combine both the statements then only of the following formats : a80, a2b26, a8b8,
64 is a number that is divisible by only one a2b2c8, a2b2c2d2
prime number and it is a perfect square hence From statement (1) alone N14 has 3 prime
by using both the statements we can answer factors hence only possibility is N = a2b2c8
the question. hence this statement is sufficient enough.
Buy books : http://www.dishapublication.com/entrance-exams-books/mba-exams.html
28
From statement (2) alone N2 has maximum (3r +1) … etc hence if N3has 112 factors then
possible number of factors, N2 must be one only possibility is that 112 = 4 x 4 x 7 or N3
of the following formats a160, a4b52, a16b16, = a3b3c6 and from this statement we can find
a4b4c16, a4b4c4d4 for maximum possible required ratio.
factors N2 = a4b4c4d4 from this we can find From statement (2) alone:- N2must be in the
out the number of factors of N3 hence this form of a2p x b2q x c2r… etc. and its number
statement alone is sufficient enough. of factors is given by (2p + 1)(2q + 1)(2r +1)
27. (c) From statement (1) alone we can conclude hence in this case N may be a2b4c4 or a22b2
that highest power of 2 is 2 but from this we hence this statement alone is not sufficient
do not know about power of 3. Hence this enough.
statement alone is not sufficient enough. 31. (e) From statement (1) alone since N/2 has 12
From statement (2) alone we can only factors hence it must be in one of the following
conclude that highest power of 3 in N is 2 formats a11, ab5, a2b3, a1b1c2

hence this statement is not sufficient enough. If N = a11 the 2N may be 212 or 21a11

If we combine both the statements then highest If N = a1b5 then 2N may be 21a1b5 or 22b5 or
power of 2 and 3 is 2 hence number must be in a1b6
the form of 22327p… since highest power of 2 If N = a2b3 then 2N may be 21a2b3 or 23b3 or
and that of 3 is same hence number of factors a224
divisible by 2 and by 3 must be same, so we If N = a1b1c1 then 2N may be 2a1b1c1 or 22b1c1
can answer that Number of factors divisible So from this statement alone we can not say
by 2 is not more than that divisible by 3. anything about N.
28. (a) From statement (1) alone we can say that From statement (2) alone definitely we cant
number is in the form of N = 22 x 7p x….. find factors of N
Number of factors of N will be 3(P+1) x ….. If we combine both the statement (1) and (2)
And 2N = 23 x 7p x….. and its number of then we have N = 212 or 23b3 hence we cant
factors are 4 x (P + 1) x…. = (81/3)4 = 27 x 4 answer.
= 108 hence this statement alone is sufficient
32. (c) From the given information N must be in the
enough. form of a17 or a1b8 or a2b5 or a1b2c2
From statement (2) alone we don’t know From statement 1 alone N is divisible by
whether N is divisible by 2 or not hence we 32 = 25 but not by 512 or 29 hence N may
cant determine the number of factors of 2N be in the form of a128 or a225 but in both the
29. (b) Since N has 16 factors hence N must be in cases we don’t know a hence this statement
one of the following formats- a15, a1b7, a3b3, alone is not sufficient enough.
a1b1c3, a1b1c1d1 Statement (2) is definitely not sufficient
From statement (1) alone N is divisible by 64 enough, if we combine both the statements
hence minimum power of 2 will be 6, then in then we will get a =5 then N = 2851 or 2552
this case we have two numbers that satisfy but in both the cases it is not divisible by 18.
this condition 215 and a1 × 27 hence from this
33. (c) None of the statement alone is sufficient
statement alone we can not find the value of N. enough. From 2nd statement number of factors
From statement (2) alone number is divisible of P is 6Q2 + 5Q + 1 = (2Q + 1)(3Q + 1) hence
by 72 or 2332 hence only possibility is that the
p must be in the form of a2Qb3Q = (a2b3)Q,
number is in the format of 23x33 hence this
Now if we combine statement (1) then we will
statement alone is sufficient enough.
get N = a2b3 and its number of factors is 12.
30. (a) From statement (1) alone:- a number N3 must
34. (b) Let N is in the form of 2paqbr…. then number
be in the form of a3p x b3q x c3r x… and its
of even factors are p(q + 1)(r + 1)… = 17 only
number of factors is given by (3p + 1) (3q + 1)
possibility is that p = 1 and q + 1 = 17 or p = 17
Buy books : http://www.dishapublication.com/entrance-exams-books/mba-exams.html
29
Hence N must be in the form of 21a16 or in the
38. (b) From statement (2) alone if k is not a factor
form of 217 hence we can not find out number of 120 then k > 6 as 120 is divisible by all
of odd factors so this statement alone is not the numbers < 7. Hence this statement alone
sufficient enough to find the answer. is sufficient enough to answer the question.
From statement (2) alone :- Let 2N is in the 39. (d) From statement (1) alone if a number has
form of 2xaybz then number of odd factors is 16 factors then minimum such number is
(y + 1)(z + 1) = 17, and that is equal to number 2 × 3 × 5 × 7 =210 hence no such two digit
of odd factors of N. factor exist.
35. (e) Let N is in the form of 2p5qarbs…. Number From statement (2) alone minimum number is
of factors that has unit digit 5 are all the odd again 2 × 3 × 5 × 7 = 210 hence this statement
factors divisible by 5. Number of factors of N alone is sufficient enough.
that has unit digit 5 = q(r + 1)(s + 1) … 40. (e) From statement (1) alone if it has only three
From statement (1) alone (q +1)(r +1)(s +1) prime factors then number of factors may
… = 21 here we have three possibilities. N = be 27 if number is in the form of a2b2c2 or
2p520 or 2p52a6 or 2p56a2 from this number otherwise more than 27 hence we can not
of factors that has unit digit 5 is given by 20, answer the question from this statement alone.
14 or 18 hence this statement alone is not From statement (2) alone we can not be sure
sufficient enough. hence this statement is not sufficient enough.
From statement (2) alone:- we can only From both the statement again number may
conclude that N is not divisible by 2 or p = 0, be in the form of a2b2c2 (as it is given that If
from this also we cant find out the required N is a perfect cube then it is a perfect square
number of factors. also but it is not given that vice-versa is also
If we combine both the statement then also true)
we have three values 20, 14 and 18. 41. (e) From statement (I) alone the pairs are
36. (e) In this case we need to find the number of (1, 30), (2, 15) (3, 10) and (5, 6) now from
co-prime factors of N, From both the statement 2nd condition we may short the pairs as (1, 30)
we don’t know whether N is divisible by 7 or and (3, 10) but from this we can not conclude
not hence even by using both the statement we a definite relation between A and B, hence this
can not answer the question. statement alone is not sufficient enough.
37. (c) Let number is 2pbqcr… then number of factors If LCM of A and B is 30 then possible pair
is (p + 1)(q + 1)(r + 1), is (1, 30), (2, 15) (3, 10) (5, 6) (2, 30) (3, 30)
Number of even factors = p(q +1)(r +1) (5,30) (6, 30) (10, 30) and (15, 30)
Number of odd factors = (q +1)(r +1) Now if AB is odd then A must be odd, this
Number of even factors is more if p > 1. statement alone not sufficient enough as in
From statement (1) we are sure that minimum (1, 30) here A = 1 and B =30 while in (2, 15)
value of p is 1 but we cant say p >1 or not A = 15 and B = 2.
hence this statement alone is not sufficient Even by using both the statement we can not
enough. find relationship between A and B.
From statement (2) alone we can only say 42. (c) From statement (I) alone, two numbers A = 4x
that number is perfect square hence it is not and B = 4y, A + B = 4(x + y) and A – B = 4(x – y)
sufficient enough. in this case since we don’t know the HCF of
If we use both the statement then we will get (x + y) and (x – y) so we can not find HCF of
minimum value of p as 2 and hence number of A and B.
even factors will be more than number of odd From statement (II) alone, we can not
factors. conclude any thing.
Buy books : http://www.dishapublication.com/entrance-exams-books/mba-exams.html
30
If we combine both the statements then LCM If we try to combine both the statement
= 4xy = 36 or xy = 9 so possible pair of (x, y) = together then we will get data inconsistent as
(1, 9) another pair (3, 3) is ruled out as A cant if one of the number is 15 then LCM can not
be equal to B so possible value of A + B is 40 be 40.
and corresponding value of A – B is 32 and 47. (e) Statement (I) alone is not sufficient enough,
their HCF is 8. we may take two examples for the values of
43. (c) From statement (I) alone, The pairs (A, B) is (A, B, C, D, E,F ) as ( 3, 4, 6, 5, 2 and 8)
(3, 16), (6, 16), (12, 16) (16, 24) and (F, 48) in this case required HCF is 8 but if we take
here F is a factor of 48, From this we can not a different case as each of them is prime
conclude about HCF of A and B. numbers then required HCF is 1.
From statement (II) alone numbers are in the From statement (II) alone, it is given that A
ratio of 1:2 so numbers can be (1, 2) or can is co-prime to D but may or may not be co-
be (8, 16) so we cant say anything about their prime to E and F hence from this statement
HCF. also we can not conclude the answer.
If we use both the statements then only From both the statement together also we can
possible pair is (24, 48) and their HCF is more not conclude the answer.
than 4. 48. (a) From the given condition if we assume f (1) = hx
B
44. (d) From statement (I) alone since A and B are and f (2) = hy, here h is the HCF of f (1) and
co-prime to each other that means A and B has f (2) then from the given condition f (3) = hxy,
no common prime factors hence their HCF is f (4) = hxy and so on each term is hxy.
1, so statement (I) alone is sufficient enough From statement (I) alone we can be sure that
to answer the question. f (10) = f (7) = 36.
From statement (II) alone similar explanation From statement (II) alone since we don’t
as above this statement also sufficient enough know the value of f (2) hence we can not find
to answer the question. the value of f (10)
45. (b) From statement (I) alone we can conclude that 49. (b) From statement (I) above we can only
number is in the form of 12K+1 but we cant conclude that a is an odd number.
find unique remainder when it is divided by From statement (II) alone we can conclude
9, hence this statement alone is not sufficient that there must be a prime common factor
enough. exist hence HCF must be more than 1. Hence
From statement (II) alone minimum such this statement alone is sufficient enough.
number is [{(1 × 5 +1) × 4 +1} × 3 + 1] = 76 50. (b) From statement (I) alone possible pairs are
and any number that satisfy this condition (1, 18) (2, 18) (3, 18) (6,18) (9, 18) (2, 9) (6, 9)
must be in the form of (3 × 4 × 5 × 6)K + 76 = Since given that one of the number is prime
360K +76 and any number in this form when hence possible pairs are (2, 18)(3, 18)(2, 9)
divided by 9 will always gives remainder but in each case they are not co-prime to
4, hence this statement alone is sufficient each other hence statement (I) alone is not
enough. sufficient enough
46. (e) From statement (I) alone possible pairs are From statement (II) alone since number of
st
(5, 8) and (5, 40) in 1 case ratio between factor is 1 only in the case if number is 1,
LCM and HCF is 40 (which is more than 8) hence their HCF is 1 or they are co-prime to
nd
but in 2 case it is 8 (Not more than 8) hence each other. So statement (II) alone is sufficient
this statement alone is not sufficient enough. enough.
From statement (I) alone we can not get a 51. (d) From statement (I) alone N is the total number
unique value of 2nd number/LCM, hence this of students then N/4 gives remainder 2 and
statement alone is not sufficient enough. N/11 gives remainder 9. Hence N must be in
Buy books : http://www.dishapublication.com/entrance-exams-books/mba-exams.html
31
the form of: k(L.C.M of 4, 11) – 2 = 44k-2 if Maximum value of N is HCF of 360 and 600
k = 1 then N = 42, if k = 2 then N = 86 hence is 120, so maximum value of R is 15, hence
this statement alone is not sufficient enough. we can not answer that R is more than 12 or
From statement (II) alone N is the total not.
number of students then N/5 gives remainder 55. (d) From statement (I) alone HCF of A, B, C and
2 and N/7 gives remainder 0 7 is the smallest D is same as HCF of 20 and 35 i.e 5, hence
number that satisfy above conditions. Hence this statement alone is sufficient enough.
N must be in the form of: k(L.C.M of 5, 7) From statement (II) alone, HCF of A, B, C and
+7 = 35k + 7 if k =1 then N = 42 and if k=2 D must be 10 hence this statement is sufficient
then N = 77, hence this statement alone is not enough.
sufficient enough. 56. (d) From statement (I) alone only possibility
If we use both the statements together then we of B and C is 1 & 25 and if any one of the
will get N = 42 will satisfy all the condition. number is 1 then required HCF is 1, hence this
52. (b) We don’t know here that the given year is a statement alone is sufficient enough to answer
leap year or not. So here we have two cases:- the question.
Case (1) :- if it is leap yaer then March will be From statement (II) alone, we will find
from 61st day to 91 day. out maximum possible value of HCF then
Case (2) If it is a non leapyaer then March possible pair of A, B, C & D is ( 25, 15, 50 &
will be from 60th day to 90th day. 75) or (10, 25, 50, 75) in both the cases HCF
From statement (I) alone both of them will is 5, hence their HCF is not more than 5, this
come together after an interval of 12 days statement alone is also sufficient enough.
i.e 12th, 24th and so on, so they will come 57. (d) From statement (I) alone, as per the given
together on 60th, 72nd and 84th day. information number must be in the form of
If it is a leap year then in a March they will 16p + z and when it is divided by 8 remainder
come on 2 days (72nd and 84th) is y hence z = 8 + y,
If it is a non leap year then in a March they Since number when divided by 8 gives
will come on 3 days (60th, 72nd and 84th) remainder y hence it must be in the form of
Hence this statement is not sufficient enough 8q + y and remainder when it is divided by 4
to give the exact answer. is x hence y = 4 + x
From statement (II) alone both of them will Hence z = 12 + x and y = 4 + x
come together after an interval of 21 days In this case possible pairs of (x, y z) = (1, 5, 13)
i.e 21st, 42nd and so on, so they will come (2, 6, 14) (3, 7, 15) in each case (x + y + z)/3
together on 63rd, 84th. remainder is 1.
Here in both the cases they will meet twice So this statement (II) alone we can only
in the month of March. Hence this statement conclude that at least 2 of the three is divisible
alone is sufficient enough. by 3 but we can not get a definite information
53. (e) about third number. Such example is (x, y, z)
54. (a) From statement (I) alone, It is HCF type = ( 4, 3, 9) in this case x + y + z is not divisible
question, here N must be a factor of by 3 and gives remainder 1 but if we take (x, y,
612 – 252 = 360 and 1212 – 612 = 600, z) as (5, 3, 9) then (x + y + z) gives remainder
Maximum value of N is HCF of 360 and 600 2 when divided by 3, hence this statement
is 120, so maximum value of R is 12, hence alone is not sufficient enough to answer the
we can answer that R is not more than 12. question.
From statement (II) alone, It is HCF 58. (d) From statement (I) alone Minimum HCF of
type question, here N must be a factor of AB and BC is B hence if HCF of (AB, BC and
615 – 255 = 360 and 1215 – 615 = 600, AC) is 1 that means B is co-prime to AC or to
Buy books : http://www.dishapublication.com/entrance-exams-books/mba-exams.html
32
A and C, similarly A & C are co-prime to each Hence this statement is not sufficient enough.
other hence they all are co-prime to each other From statement (I) alone, a must be an odd
and their HCF is 1. number but not divisible by 5 hence not ends
From statement (II) alone similar explanation with 5, then condition is similar to previous
as above A, B and C are co- prime to each case except case 3 (i.e 4, 6) hence unit digit
other. of LCM is 0, so this statement is sufficient
59. (d) Let F(1) = hx, and F(2) = hy then enough.
F(3) = h2xy, F(4) = h2xy, F(5) = h2xy, 61. (e) From statement (i) alone we have following
F(6) = h4x2y2, F(7) = h4x2y2, F(8) = h4x2y2 cases:-
F(9) = h8x4y4, F(10) = h8x4y4, F(11) = h8x4y4 (x, y) = (2,3)
and so on. xy -> 23 -> 8 or 32 -> 9
Hence required HCF is same as F(3n) We will not get unique value of unit digit
From statement (I) alone since F(3n + 9) = hence statement (i) alone is not sufficient
1248, hence F(3n + 6) = 1224, F(3n + 3) = 1212 enough.
and F(3n) = 126, hence this statement alone is Statement (ii) alone y is a multiple of 10.
sufficient enough.
If we use both the statement then also we will
From statement (II) alone, since p>n so
not get a unique value of unit digit.
3p-6 > 3n-6 and hence F(3n-6) and F(3p-6) is
62. (b) We know that cyclicity of unit digit of 7 is
F(3n-6) = 1224 then F(3n-3) = 1248 and F(3n)
96 four hence we will divide the given power by
= 12 . This statement alone is sufficient
4 if remainder is 1, 2, 3 and 0 then unit digit is
enough.
2 same as that of 71, 72, 73, or 74.
60. (b) From statement (I) alone, if a divided by 8
remainder is 1 hence a must be odd number, So in this question we have to find the
so A and B are two consecutive even numbers remainder when ‘abcd’ is divided by 4 i.e
then their HCF must be 2 and hence their when two digit number ‘cd’ divided by 4.
LCM will be (AB)/2 so unit digit of LCM From statement (i) alone two digit number
will be unit digit of (AB)/2 since A and B are ‘ad’ is divisible by 4. Or we can say that d
two consecutive even numbers hence unit must be an even number. This statement alone
digits of A & B must be (0, 2), (2, 4), (4, 6), is not sufficient enough.
(6, 8), (8, 0) From statement (ii) alone since ‘abcd’ divided
If a ends with 1 then a4444 ends with 1 and by 72 it leaves remainder 1 hence when it is
unit digit of (A, B) = (0, 2) and unit digit of divided by 4 it leaves remainder 1 and so
(AB)/2 is 0 required unit digit is 7. Statement (ii) alone is
If a ends with 3 then a4444 ends with 1 and sufficient enough.
unit digit of (A, B) = (0, 2) and unit digit of 63. (b) Since cyclicity of unit digit of 3 is 4 hence we
(AB)/2 is 0 will divide 3x by 4, we know that 3odd divided
If a ends with 5 then a4444 ends with 5 and by 4 remainder is 3 and 3even divided by 4
unit digit of (A, B) = (4, 6) and unit digit of remainder is 1, hence what all we have to find
(AB)/2 is 2 is that x is even or odd.
If a ends with 7 then a 4444 ends with 1 and From statement (i) x may be even (if = 2)
unit digit of (A, B) = (0, 2) and unit digit of otherwise it is odd hence this statement is
(AB)/2 is 0 alone not sufficient enough.
If a ends with 9 then a 4444 ends with 1 and From statement (ii) alone x is a three digit
unit digit of (A, B) = ( 0, 2) and unit digit of prime number hence it must be odd hence this
(AB)/2 is 0 statement alone is sufficient enough.
Buy books : http://www.dishapublication.com/entrance-exams-books/mba-exams.html
33
64. (c) If a number has only 2 factors then it must be 67. (e) From statement (i) alone we can say that unit
a prime number. And we know that 4odd has digit of a ends with either 1 or with 9. Hence
unit digit 1 and 4even has unit digit 6. this statement alone is not sufficient enough.
From statement (i) alone, x –1 is prime number We can not conclude anything from statement
P then x = P + 1 if P is 2 then x = 3 (odd) or (ii) alone.
if P is any other number than 2 (say 3) then If we use both the statements then we can say
x = odd hence this statement is not sufficient that a is a multiple of 9 that ends with 9, so
enough. even by using both the statements we can not
From statement (ii) alone, x + 1 is prime find the unique value of a.
number P then x = P – 1, again as in the 68. (b) From statement (i) alone since 66 is in the
previous case x may be even or may be odd, form 4k +2 hence unit digit of a66 is same as
hence this statement alone is not sufficient that of a2 hence unit digit of a is either 1 or 9.
enough. Hence this statement alone is not sufficient
If we use both the statements then x – 1 and enough.
x + 1 both are prime numbers hence that prime From statement (ii) alone since 999 is in the
number can not be 2, (such pairs of prime form of 4k + 3 hence unit digit of is same as
numbers are 3 & 5 when x = 4, 5 & 7 when that of which is given 9 hence unit digit of a
x = 6) hence x must be even, so by using both must be 9 and hence required unit digit of is 9
the statements we cans say that x is even and so statement (ii) alone is sufficient enough.
required unit digit is 6. 69. (b) From statement (i) alone since 333 is in the
65. (a) We know that cyclicity of unit digit of 2 is 4 form of 4k + 1 hence unit digit of a333 is
hence we will divide the given exponent by 4. same as that of a hence a = 9 since we don’t
From statement (i) alone since a, b, c and d know the value of b hence we can not find the
are even numbers hence must be divisible by required unit digit.
4, so this statement is sufficient enough to From statement (ii) alone since 555 is in the
answer the question. form of 4k + 3 hence unit digit is same as that
From statement (ii) alone is in the form of of a3 since it is given 6 hence unit digit of a
odd odd when this is divided by 4 remainder must be 6 and hence required unit digit is 6.
may be 1 or 3 hence this statement alone is not Statement (ii) alone is sufficient enough.
sufficient enough to answer the question. 70. (d) We know that unit digit of 9odd is 9 and 9even
66. (e) Since cyclicity of unit digit of 3 is 4 hence is 1.
we will divide the given power by 4 and its From Statement (i) alone since x + y is odd
remainder will decide the unit digit. hence one of them must be even and one of
From statement (i) alone we don’t know them must be odd and one of them is even,
whether a is even (2) or odd (other prime hence required unit digit is given by 9odd +
numbers) hence this statement is not sufficient 9even has unit digit 1+ 9 = 10 or required unit
enough. digit is 0. Statement (i) alone is sufficient
From statement (ii) alone we can not conclude enough.
anything as three digit number may be even or From statement (ii) alone since product x.y
odd. is odd hence both x and y must be odd hence
If we use both the statements then we can 9odd + 9odd or required unit digit is 8, statement
conclude that number is a three digit prime (ii) is alone sufficient enough.
number that means odd number, but we don’t 71. (d) Since two digit number “xy” divisible by 4
know remainder when given exponent divided hence y must be even i.e. (0,2,4,6,8)
by 4. If Y=0 then unit digit will be 0
Buy books : http://www.dishapublication.com/entrance-exams-books/mba-exams.html
34
Y=2 then unit digit given by 24k which is 6 If x =1 then 17 + 71 has unit digit 8
Y=4 then unit digit given by 44k which is 6 If x =3 then 37 + 73 has unit digit 0
Y=6 then unit digit given by 64k which is 6 If x =5 then 57 + 75 has unit digit 2
Y=8 then unit digit given by 84k which is 6 If x =7 then 77 + 77 has unit digit 6
Hence unit digit can be either 0 or 6. So
If x =9 then 97 + 79 has unit digit 6
statement (i) is not sufficient enough.
From statement (ii) alone (xy)xy is not From statement (i) alone summation is 6
divisible by 100, so from this statement we which can be given by two values of x = 7 & 9
cant conclude anything. hence this statement is not sufficient enough.
If we use both the statements then we will get From statement (ii) alone we will get x = 1.
unit digit 6. 75. (e) From statement (i) unit digit is 3 hence we
72. (e) We know that 34k+1 has unit digit 3, 34k+2 has have following cases if unit digit of x is 3
unit digit 9, 34k+3 has unit digit 7, and 34k has then y should be in the form of 4k + 1, if unit
unit digit 1 digit of x is 7 then y should be in the form of
Unit digit 0 is given by either 3 + 7 or 9 + 1 4k + 3, from this statement we cant say
Hence we have two cases – anything about unit digit of y hence this
Case (i) if 3 & 7 then x and y must be in the statement alone is not sufficient enough.
form of 4p + 1 and 4q + 3, then From statement (ii) alone unit digit 1 then we
xy = (4p + 1)(4q + 3) = 16pq + 12p + 4q + 3 have following cases if unit digit of y is 1 then
hence when xy divided by 4 remainder is 3. x can be any number, if unit digit of y is 3 then
Case (ii) if 1 & 9 then x and y must be in the x is in the form of 4k, if unit digit of y is 7
form of 4p and 4q + 2, then xy = (4p)(4q + 2) then x is in the form of 4k, if unit digit of y is
hence when xy divided by 4 remainder is 0. 9 then x is in the form of 2k, as in the previous
From statement (i) alone x may be in the case we cant conclude anything hence this
form of either 4k or 4k + 1 hence we can not statement alone is not sufficient enough.
conclude whether it is in case (i) or in case (ii)
If we use both the statement then we also we
hence this statement is not sufficient enough.
cant conclude anything
From statement (ii) alone x may be in the form
of either 4k + 2 or 4k + 3 hence we can not 76. (b) From statement (i) alone x is in the form of
conclude whether it is in case (i) or in case (ii) 30k-1 and cyclicity of last two digit of 2 is
hence this statement is not sufficient enough. 20 and a number in the form of 30k-1 divided
Even if we use both the statements we can by 20 will give us remainder 10k-1 which
not find the unique value of remainder when is either 9 or 19 hence this statement is not
xy divided by 4 hence even by using both sufficient enough.
the statements we can not find the value of From statement (ii) alone x is in the form of
required unit digit. 20k-1 and any number in this format when
73. (a) Since unit digit of 3 has cyclicity of 4. divided by 20 will always give remainder 16
From statement (i) alone unit digit of x is 3 hence this statement is sufficient enough.
and that of y is 7 but y > x hence unit digit is 4,
77. (c) We know that unit digit of the given expression
so statement (i) alone is sufficient enough. is 1 and 2nd last digit is given by unit digit
From statement (ii) alone if x > y then unit of product of 8 and z, so we have to find the
digit is 6 and if x < y then unit digit is 4 hence value of z.
this statement alone is not sufficient enough. From statement (i) alone xyz is a multiple of
74. (b) In both the statements summation is even that 35 hence we can not find a unique value of z
is possible when x is odd, we have following (it may be 0 or 5) hence this statement alone
cases- is not sufficient enough.
Buy books : http://www.dishapublication.com/entrance-exams-books/mba-exams.html
35
From statement (ii) alone xyz is a multiple of 81. (d) Since cyclicity of remainder of [3/7x] is 6,
115 hence we can not find a unique value of z hence we have to divide x by 6 and find the
(it may be 0 or 5) hence this statement alone remainder. From statement (i) alone when x is
is not sufficient enough. divided by 6, we won’t get a unique remainder.
If we use both the statement then we will Similarly from statement (ii) alone we
get xyz a multiple of 5, 7 and 23 only three cannotget a unique remainder. If we use both
digit number that satisfy this condition is 805. the statement together, then from L.C.M. type
Hence both the statement require to answer 2 x must be in the form of P (L.C.M. of 324)
the question. - 2 i.e. x = 12 P-2. When any number in this
78. (a) Since 76^n has last two digit 76 hence format divided by 6 will give us remainder
statement (i) is sufficient enough to answer -2 (or 4). Final remainder is same as that of
the question. 4th point of cycle or that of [34/7] which is
79. (c) From statement (i) alone ab has unit digit 1, 4. Hence both the statements are required to
hence possible cases are when a = 1 then b can answer the question.
be any digit, when a =3, then b can be 4 or 8, 82. (b) Cyclicity of remainder of [5x/9] is 6. From
when a = 7, then b = 4 or 8, when a =9 then statement (i) alone 2 digit prime no is in the
b = 2, 4, 6 or 8 hence from this statement form of 6k ± 1 hence we cannot find remainder
alone we can not conclude anything. when x divided by 6, so this statement alone is
From statement (ii) alone since unit digit of not sufficient enough.
ba is 4 hence possible cases are, when b =2, From statement (ii) alone number is in the
then a = 2 or 6, when b = 4 then a = 1, 3, 5, 7 form of 12k + 9 ; it is when divided by 6 will
or 9, when b = 8 then a = 2 or 6. Hence this give remainder 3. Hence this statement alone
statement alone is not sufficient enough. is sufficient enough.
If we use both the statement together then we 83. (e) From statement (i) alone a is in the form of
will get possible pairs that satisfy both the 5x + 1 and we can’t find remainder when it
cases are (a, b) = (1, 4), ( 3, 4), (7, 4) or (9, 4) is divided by 7. Hence, this statement is not
Since difference between ab and ba is not sufficient enough similar to statement (i).
divisible by 27 and |a – b| ≠ 3 hence (1, 4) and 84. (e) Since cyclicity of remainder of division
(7, 4) ruled out. So possible pairs of (a, b) is [7^k /9] is 3 hence to get desired remainder
(3,4 ) or (9,4) we have to find the remainder when k divided
Last two digit of 3443 is 04 by 3 but here k = ab, so we have to find the
Last two digit of 9449 is 04, [At this point remainder when ab is divided by 3.
when we are at the end of this chapter please From statement (i) alone since a and b both are
don’t wonder how last two digits of 3443 and prime numbers so we cannotfind remainder
9449 is 04] when ab is divided by 3.
Hence by using both the statement we can be From statement (ii) alone since given that a
sure that required last two digit is 04 and b both are even numbers then also we
80. (e) From statement (i) alone if a = 3 then b = 6 cannotfind the remainder when ab divided by
and if a =9 then b =3 hence this statement 3.
alone is not sufficient enough. Even by using both the statements we cannot
From statement (ii) alone if a =4 then b = 4 find the required remainder. x
85. (c) From statement (i) alone remainder of   is
and if a =2 then b = 8 hence this statement z
alone is not sufficient enough. ±1 hence if y is even then remainder is 1 and
Even by using both the statements we cant if y is odd then remainder is –1. Hence this
answer the question. statement alone is not sufficient enough.
Buy books : http://www.dishapublication.com/entrance-exams-books/mba-exams.html
36

 x3  89. (a) The required remainder is same as that of ‘x’


From statement (ii) alone remainder of  
 z  ( ( ))
 y z −1 
 x 
is 1 but since we don’t know the remainder time of remainders of  
 y 
when z divided by 3 hence this statement
alone is not sufficient enough. From statement (i) alone the remainder of
If we use both the statements together then we
x
( ( ))
 y z −1 
 x 
can be sure that remainder of   is 1 hence   is 1 hence actual remainder is
 z   y 
required remainder is 1,  x2  x. since we don’t know the exact value of x
86. (e) From statement (i) alone remainder of  
 z  hence this statement alone is not sufficient
is 1 from this we can conclude that remainder enough.
of [x/z] may be +1 or –1, hence this statement From statement (ii) alone again we cannotfind
alone is not sufficient enough.  x3  the required remainder.
From statement (ii) alone remainder of   If we use both the statements then we will get
 z  x = 6 and y =5
is 1 from this we can conclude that remainder
90. (e) From statement (i) alone since x + y + z = 25,
of [x/z] is 1hence this statement alone is
possible values of x, y and z is 9, 9, 8 in any
sufficient enough to answer the question.
order hence we cannotget a unique remainder.
 3k 
87. (a) Cyclicity of remainder of   is 6- From statement (ii) alone we cannotconclude
7 anything.
From statement (i) alone since x = 6y hence If we use both the statements then we will not
both the remainders are same i.e R1 = R2 get any value of x, y z that satisfy both the
hence this statement alone is sufficient enough conditions.
to say that R1 is not more than R2. x
From statement (i) alone since x = 4y, so in 91. (d) Let remainder of  7  is R then
 xk 
some cases R1 is more than R2, while in some Let R=1, then cyclicity of remainder of  
cases less and in some cases even it may be 7 
equal, hence this statement is not sufficient is 1. Hence y can be any integer.
 xk 
enough.  2x  Let R=2, then cyclicity of remainder of  
88. (c) Since cyclicity of remainder of   is 3 and is 3. Hence y is in the form of 3p 7 
7 
3 
y  xk 
remainder of   is 6, hence we have to Let R=3, then cyclicity of remainder of  
7  is 6. Hence y is in the form of 6p 7 
find the remainder when x and y divided by 3
 xk 
and 6 respectively. Let R=4, then cyclicity of remainder of  
From statement (i) alone we cannot find the is 3. Hence y is in the form of 3p 7 
remainder when y divided by 6, hence this  xk 
statement alone is not sufficient enough. Let R=5, then cyclicity of remainder of  
From statement (ii) alone we cannotfind the is 6. Hence y is in the form of 6 p 7 
remainder when y divided by 6, hence this  xk 
statement alone is not sufficient enough. Let x = 6, then cyclicity of remainder of  
is 2. Hence y is in the form of 2p 7 
If we use both the statements then we can say
that remainder when x divided by 3 remainder Hence if y is a multiple of 3 then R can be any
is 0 and when y divided by 6 remainder is 3, integer from 1 to 6, if y is a multiple of 6 then
hence by using both the statements we can R can be either 3 or 5. If y is a multiple of 2
answer the question. then R is 6.
Buy books : http://www.dishapublication.com/entrance-exams-books/mba-exams.html
37
From statement (i) alone since y is a two digit If we use both the statements then we will get
prime number hence it is not divisible by 2 remainder 1 from Wilson’s theorem.
and 3 so we must have case 1 and R =1, hence 95. (b) From extension of Wilson’s Theorem-
this statement alone is sufficient enough. From statement (i) alone remainder is 2x,
From statement (ii) alone since y is a number since we don’t know the value of x, hence this
which is not divisible by any digit so we must statement is not sufficient enough.
have case 1 and R =1, hence this statement From statement (ii) alone remainder is 1, hence
alone is sufficient enough. this statement alone is sufficient enough.
92. (e) From statement (i) alone since remainder 96. (e) This question is exactly same as that
of question number 20 of section Data
of [x/6] is 4 hence x must be in the form of
Sufficiency of Chapter 5.
6k + 4 from this statement alone we cannotfind
From statement (i) alone if a = 3 then b = 6
the remainder when x divided by 24, hence and if a =9 then b =3 hence this statement
this statement alone is not sufficient enough. alone is not sufficient enough.
From statement (ii) alone since remainder of From statement (ii) alone if a = 4 then b = 4
[x/4] is 3 hence x must be in the form of 4k +3 and if a =2 then b = 8 hence this statement
from this statement alone we cannot find the alone is not sufficient enough.
remainder when x divided by 24, hence this Even by using both the statements we cannot
statement alone is not sufficient enough. answer the question.
If we use both the statement, actually we 97. (c) From statement (i) alone we cannotfind
cannotuse both the statements simultaneously remainder when a divided by bc, hence this
as they are not compatible (means from statement alone is not sufficient enough.
statement (i) x is even but from statement (ii) From statement (ii) alone we cannotfind
x is odd) hence we cannotfind the required remainder when a divided by bc, hence this
answer even by using both the statements. statement alone is not sufficient enough.
93. (a) From statement (i) alone x must be in the form If we use both the statements then we will get
of k(L.C.M of 18 & 8) + 1 or in the form of b =2 (the only even prime number), and a is
72k + 1 and any number in this format when a composite even hence minimum value is
divided by 36 given remainder 1, hence final 4, hence is always divisible by, and required
remainder is zero.
remainder is 1 and it is independent from
98. (c) From both the statements we can find that
y. hence this statement alone is sufficient
y =10, then x =100 and then denominator is
enough. xa + yb + c =100a + 10b + c = three digit
From statement (ii) alone x must be in the number ‘abc’. In that case remainder is 0.
form of k(LCM of 18 & 8) - 1 or in the form of Hence by using both the statements we can fid
72k-1 and any number in this format when the required remainder.
divided by 36 given remainder -1 (or 35), 99. (a) From the extension of Wilson’s theorem we
hence final remainder is 1 when y is even know that-
and it is -1 or 35 when y is odd, hence this If 2n + 1 is a prime number then
statement alone is not sufficient enough. Case (1) when n is odd then
94. (c) We know from Wilson’s theorem that
 ( P − 2 ) ! Remainder of 
{ }
 ( n !)2 
 is 1
remainder of   is 1 here P is a prime  2n + 1 
number.  P 
Case (2) when n is Even then
From statement (i) alone we cannotfind the
required remainder, Remainder of 
{ }
 ( n !)2 
 is 2n
From statement (ii) alone we cannotfind the  2n + 1 
required remainder, Hence statement (i) alone is sufficient enough.
Buy books : http://www.dishapublication.com/entrance-exams-books/mba-exams.html
38
100. (d) From statement (i) alone since a, b and c is From statement (ii) alone a is divisible by
a two digit number hence unit digit of their 6 and hence this statement alone is also
factorial must be 0, and hence required unit sufficient enough.
digit is 0, so this statement alone is sufficient 105. (a) We know that a! is divisible by 64 if a ≥ 17
enough. From statement (i) alone if a! is divisible by
From statement (ii) alone a, b and c is a prime 17 then a ≥ 17, hence this statement alone is
number except 2 and 3 hence unit digit of sufficient enough.
their factorial must be 0 and hence required From statement (ii) alone if a! is divisible by
unit digit is zero. Hence statement (ii) alone is 26 then a ≥ 13, hence this statement alone is
sufficient enough. not sufficient enough.
101. (a) From statement (i) alone since a ≥ b hence a! 106. (b) From statement (i) alone since highest power
must be divisible by b! and hence remainder is of 9 in a! is 5 hence that of 3 must be either 9
or 10
zero, this statement is sufficient enough.
Highest power of 3 in 24!, 25! And 26! Is
From statement (i) alone since a ≤ b if a = b
10 and that in 21!, 22!, and 23! Is 9 so this
then remainder of [a!/b!] is zero and when statement alone is not sufficient enough.
a < b then remainder depends on the value From statement (ii) alone since highest power
of a and b, hence this statement alone is not of 9 in a! is 6 hence that of 3 must be either 12
sufficient enough. or 13
102. (c) From statement (i) alone 63 ≤ N < 72, if Highest power of 3 in 24!, 25! And 26! Is 10
63 ≤ N <70 then highest power of 7 is 10 and that in 27!, 28!, and 29! Is 13, so only
and if 70 ≤ N < 72 then highest power of 7 possibility is that highest power of 3 is 13, so
in N! is 11,, hence this statement alone is not this statement alone is sufficient enough.
sufficient enough. 107. (a) From statement (i) alone since highest power
From statement (ii) alone 55 ≤ N < 65, if N of 27 in a! is 12 hence that of 3 must be either
=55 then highest power of 7 is 8, if 56 ≤ N <63 36, 37 or 38
then highest power of 7 is 9 and if 63 ≤ N < 65 Highest power of 3 in 78!, 79! And 80! Is 36
then highest power of 7 in N! is 10,, hence this and that in 81!, 82! And 83! Is 40 so highest
statement alone is not sufficient enough. power or 9 must be 18 so this statement alone
If we combine both the statements then we is sufficient enough.
will get 63 ≤ N < 65 and in any case highest From statement (ii) alone since highest power
power of 7 in N! is 10, hence we can answer of 27 in a! is 9 hence that of 3 must be either
27 or 28 or 29
the question by using both the statements.
Highest power of 3 in 57!, 58! And 59! Is 27
103. (b) From statement (i) alone it is not known that
and that in 60!, 61!, and 62! Is 28, so highest
a is a two digit number or not (as factorial of power of 9 may be 13 or 14 so this statement
a two digit number will have last two digits as alone is not sufficient enough.
00) hence this statement alone is not sufficient 108. (a) From statement (i) alone minimum value of N
enough. is 5, and hence required unit digit is 0 so this
Statement (ii) alone is sufficient enough. statement alone is sufficient enough.
104. (d) If a! is divisible by 18 then minimum value of From statement (ii) alone N can be any odd
a is 6, as 6! Is the smallest factorial divisible multiple of 3 hence this statement alone is not
by 72. sufficient enough.
From statement (i) alone 6! Is the smallest 109. (a) From statement (i) alone N must be either 100
factorial divisible by 18. And in this case it is or 101 in both the cases highest power of 5
divisible by 72 as well, hence this statement and hence 10 is 24 so this statement alone is
alone is sufficient enough. sufficient enough.
Buy books : http://www.dishapublication.com/entrance-exams-books/mba-exams.html
39
From statement (ii) alone Highest power of a! but in both the cases it is more than 6 hence
2 is either 46 or 47 from trail and error we highest power of 10 is 6, so this statement
know that possible values of N is 49 and 50, alone is sufficient enough.
since highest power of 2 in 49! And 50! Is 46 From statement (ii) alone a must be either
and 47 respectively, and highest power of 5 or 48 or 49 and in both the cases highest power
10 in 49! And 50! Are 10 and 12 respectively of 5 in a! is 10 hence this statement alone is
hence so this statement alone is not sufficient sufficient enough.
enough. 114. (a) From statement (i) alone a ≥ 7 hence unit
110. (e) We know that highest power of 10 in 50! Is 12. digit of a! must be zero and hence required
From statement (i) alone N can be 49, 50, 51, unit digit is zero hence this statement alone is
52, or 53, If it is 49 then answer is 10 or else sufficient enough.
12 hence this statement alone is not sufficient From statement (ii) alone if a = 0 then b = 4
enough and then unit digit of (a!)(b!) is 4 or else unit
From statement (ii) alone N can be 46, 47, 48, digit is zero, hence this statement alone is not
49, or 50, If it is 49 then answer is 10 or else sufficient enough.
12 hence this statement alone is not sufficient 115. (b) From statement (i) alone since a = b +7 hence
enough. [a!/b!] = (b +1)(b + 2)(b + 3)… (b + 7) if
If we use both the statements together then b > 3 then last two digit will be given by
also we can notbe sure as N= 49 and 50 hence 4 x 5 x .. x 9 x 10 which has last two digit as
even by using both the statements we can not zero but if b is not more than 3 then last two
answer the question. digits is not zero, hence this statement alone is
111. (c) From statement (i) alone we can find that not sufficient enough.
roots of the given quadratic equation is 6 or -1 From statement (ii) alone if b =5k then [a!/b!]
hence a = 6 but from this statement we don’t = (5k + 1)(5 k + 2)(5k + 3)(5k + 4) which has
have any information about ‘b’ hence this last two digits 24 irrespective of the value
statement alone is not sufficient enough. of k, hence this statement alone is sufficient
From statement (ii) alone we can not find enough.
a definite value of ‘a’ and ‘b’ hence this 116. (a) From statement (i) alone 19 is the next prime
statement alone is not sufficient enough. number after 17, from the given condition
If we use both the statement then b > a and 34 ≤ N/2 < 37 or 68 ≤ N < 74, since factorial
a = 6 hence required HCF is 6!. of N/2 is defined hence N must be an even and
112. (b) From statement (i) alone since highest power possible values of N is 68, 70, 72. Whatever
of 2 in a! is 41 hence a can be 44 or 45, and the value of N may be highest value of prime
highest power of 5 in 44! Is 9 and that in 45! number that has exponent 2 in N! must be
Is 10, hence we can notfind highest power of 31, hence this statement alone is sufficient
5 in a! so this statement alone is not sufficient enough.
enough. From statement (ii) alone 13 is the next prime
From statement (ii) alone a must be either number after 11, from the given condition
48 or 49 and in both the cases highest power 22 ≤ N/3 < 26 or 66 ≤ N < 78, since factorial
of 5 in a! is 10 hence this statement alone is of N/3 is defined hence N must be divisible by
sufficient enough. 3 and possible values of N is 66, 69, 72 and
113. (d) From statement (i) alone since highest power 75. If N is 66 then largest prime number is 31
of 2 in a! is 41 hence a can be 44 or 45, and and if N is 75 then largest prime number is
highest power of 5 in 44! Is 9 and that in 45! Is 37 hence this statement alone is not sufficient
10, hence we can notfind highest power of 5 in enough.
Buy books : http://www.dishapublication.com/entrance-exams-books/mba-exams.html
40
117. (a) We know that 3293 = 37 × 89 If we use both the statement together then we
From statement (i) alone from the given can find the value of n and that of r, and hence
condition 51 ≤ N/2 < 68 or 102 ≤ N < 138, we can answer the question by using both the
since factorial of N/2 is defined hence N must statements together.
be an even. Whatever the value of N highest 120. (d) Log A X > Log A X2 as x is an integer so X2
power of 89 is 1 and that of 37 is more than 1 > X log of a greater number is less than that
hence highest power of 3293 must be 1, hence of a smaller number only when the base lies
this statement alone is sufficient enough. between 0 and 1, So both the statement I and
From statement (ii) alone from the given II are sufficient to answer. Option( D)
condition 68 ≤ N/2 < 111 or 136 ≤ N < 222,
121. (e) From statement I and II we cannot find the
since factorial of N/2 is defined hence N must
answer, when we combine, we have
be an even. In this case highest power of 89
is 1 or 2 hence this statement alone is not a2 + b 2 + 2ab = 25, (a + b)2 = 25, (a + b) = 5
sufficient enough. or –5. Still we are not having any idea.
118. (d) From statement (i) alone highest power of 9 122. (a) By using Statement I we can calculate the
in N! is 8 hence that of 3 in N! is 16 or 17, value of X, Statement Ii is redundant.
but from trail we will get highest power of 123. (c) From I, X = 1 or –2 and Y = 3 or –5
3 in 33! Is 15 and that in 36! Is 17, hence
From II, X = 4 or –2 and Y = 3 and –6
36 ≤ N < 38, and highest power of 2 in 36!
and 37! Is 34 while that in 38! Is 35. But in Combining both we get X = –2 and Y = 3
all the cases highest power of 16 is 8 and 124. (a) Let X = 2 and Y = 1, than ( X2 + Y2 ) = 5 and
number of zeros when N! is converted to base ( + ) = 1.5, hence I alone is sufficient.
16 is 8, hence this statement alone is sufficient 125. (e) For Statement I : we get X = 27 or 8, from the
enough. value of x we cannot find the unique value of
From statement (ii) alone highest power of X as both the values of x are perfect cube. So
12 in N! is 8 hence that of 3 in N! is 8 and if X is perfect cube X2 must be perfect cube.
highest power of 4 in N! is 8 or that of 2 is 16 Both the statement are not sufficient to
or 17. Possible value of N is 36, 37 or 38 in answer.
any case highest power of 16 in N! is 8 8 and
number of zeros when N! is converted to base 126. (c)
16 is 8, hence this statement alone is sufficient 127. (e) Both the statement are not sufficient to answer
enough. the question as From one X = 8 and Y = 7 than
n! X > y and from II, X = –8 and Y = –7 where
119. (c) Since nCr = Y > X.
{r ! n − r )!}
(
( r − 2 )!
From statement (i) alone nCr = 128. (a) C can take any value except ¼. For X to
{r !2!} = be real, C < ¼. Science we cannot get a
( r + 1)( r + 2 ) hence this statement alone is
unique value of C. Statement I is sufficient
but Statement II is not sufficient as no more
not sufficient enough.
information is given.

From statement (ii) alone since highest power
of 2 in n! is 11 hence n = 14 or 15 but highest 129. (d) We know X3 + Y3 + Z3 – 3XYZ
power of 3 in n! is 6 hence n must be 15. From = ( X + Y + Z) (X2 + Y2 + Z2 – XY – YZ – ZX).
this statement we don’t have any information Each statement alone is sufficient to give the
about r hence this statement alone is not answer.
sufficient enough.

Buy books : http://www.dishapublication.com/entrance-exams-books/mba-exams.html

You might also like